You are on page 1of 56

SFG 2023 |LEVEL 1 |Test 27 |

DO NOT OPEN THIS TEST BOOKLET UNTIL YOU ARE TOLD TO DO SO


T.B.C.: FIAS-SFG23-L1T27 Test Booklet Series
Serial No.

A
TEST BOOKLET
GENERAL STUDIES

Time Allowed: ONE HOUR Maximum Marks: 100


INSTRUCTIONS
1. IMMEDIATELY AFTER THE COMMENCEMENT OF THE EXAMINATION, YOU SHOULD CHECK
THAT THIS TEST BOOKLET DOES NOT HAVE ANY UNPRINTED OR TORN OR MISSING PAGES
OR ITEMS ETC. IF SO, GET IT REPLACED BY A COMPLETE TEST BOOK.
2. Please note that it is the candidate’s responsibility to encode and fill in the Roll Number and Test
Booklet Series A, B, C or D carefully without any omission or discrepancy at the appropriate
places in the OMR Answer Sheet. Any omission/discrepancy will render the Answer Sheet liable
for rejection.
3. You have to enter your Name, Email Id and
Name:
Mobile No. on the test booklet in the Box Email Id:
provided alongside. DO NOT write anything
Mobile No:
else on the Test Booklet.
4. This Test Booklet contains 50 items (questions). Each item is printed in English and Hindi. Each
item comprises four responses (answers). You will select the response which you want to mark
on the Answer Sheet. In case you feel that there is more than one correct response, mark the
response which you consider the best. In any case, choose ONLY ONE response for each item.
5. You have to mark all your responses ONLY on the separate Answer Sheet provided.
See directions in the Answer Sheet.
6. All items carry equal marks.
7. Before you proceed to mark in the Answer Sheet the response to various items in the Test Booklet,
you have to fill in some particulars in the Answer Sheet as per instructions sent to you with your
Admission Certificate.
8. After you have completed filling in all your responses on the Answer Sheet and the examination
has concluded, you should hand over to the Invigilator only the Answer Sheet. You are permitted
to take away with you the Test Booklet.
9. Sheets for rough work are appended in the Test Booklet at the end.
10. Penalty for wrong answers:
THERE WILL BE PENALTY FOR WRONG ANSWERS MARKED BY A CANDIDATE IN THE
OBJECTIVE TYPE QUESTION PAPERS.
(i) There are four alternatives for the answer to every question. For each question for which a
wrong answer has been given by the candidate, one-third of the marks assigned to that
question will be deducted as penalty.
(ii) If a candidate gives more than one answer, it will be treated as wrong answer even if one
of the given answers happens to be correct and there will be same penalty as above to that
question.
(iii) If a question is left bank i.e., no answer is given by the candidate, there will be no penalty
for that question.

Forum Learning Centre: Delhi - 2nd Floor, IAPL House, 19 Pusa Road, Karol Bagh, New Delhi - 110005 | Patna - 2nd floor, AG Palace, E Boring Canal
Road, Patna, Bihar 800001 | Hyderabad - 1st & 2nd Floor, SM Plaza, RTC X Rd, Indira Park Road, Jawahar Nagar, Hyderabad, Telangana 500020
9821711605 | https://academy.forumias.com | admissions@forumias.academy | helpdesk@forumias.academy
Page 1

SFG 2023 |LEVEL 1 |Test 27 |

Q.1) Westerlies in southern hemisphere are 2. Poor air quality in winter months.
stronger and persistent than in northern 3. Shifting of human settlements to mountain
hemisphere. Why? tops from the mountain valleys.
1. Southern hemisphere has less landmass as 4. Thriving vegetation in the Valley regions.
compared to northern hemisphere. Which of the options given above are correct?
2. Coriolis force is higher in southern a) 1, 2 and 3 only
hemisphere as compared to northern b) 1 and 4 only
hemisphere. c) 2 and 3 only
Which of the statements given above is/are d) 1, 2, 3 and 4
correct?
a) 1 only Q.5) With reference to Kalanamak Rice,
b) 2 only consider the following statements:
c) Both 1 and 2 1. This rice variety has been recently
d) Neither 1 nor 2 developed by the Indian Institute of
Sciences.
Q.2) With respect to the Air Masses, consider 2. It is a high yielding rice variety owing to its
the following statements: tall height.
1. They are very small packets of atmospheric 3. It is usually grown in the Terai region of
air showing great changes in temperature Uttar Pradesh.
and moisture. Which of the statements given above is/are
2. They are formed when wind speeds are correct?
sufficiently low. a) 1 only
3. They do not form over oceans, and can be b) 3 only
formed over lands only. c) 2 and 3 only
4. They often bring changes in the weather d) 1, 2 and 3
conditions.
Which of the statements given above are Q.6) In Northern India during summer season,
correct? local winds called ‘loo’ can be observed to be
a) 1, 2 and 4 only blown. This wind is mainly the outcome of
b) 2 and 4 only which one of the following processes?
c) 1 and 4 only a) Conduction
d) 2 and 3 only b) Radiation
c) Advection
Q.3) With reference to Isotherms, consider the d) Convection
following statements:
1. These are imaginary lines connecting the Q.7) With reference to the Coriolis Force,
places with same temperatures. consider the following statements:
2. These lines are generally parallel to the lines 1. It is responsible for the deflections in the
of longitude. direction of Winds.
3. Closely spaced isotherms indicate very 2. It acts perpendicular to the direction of
gradual variation in temperature. velocity of the object.
Which of the statements given above is/ are 3. Coriolis Force is higher at the Poles, than at
correct? the Equator.
a) 1 and 3 only 4. The latitude is the sole factor influencing its
b) 1 and 2 only strength.
c) 2 and 3 only Which of the statements given above are
d) 1 only correct?
a) 1, 2 and 3 only
Q.4) With reference to the likely effects of the b) 2 and 4 only
Inversion of Temperature, consider the c) 1 and 3 only
following: d) 1, 2, 3 and 4
1. Suppression of rainfall.

Forum Learning Centre: Delhi - 2nd Floor, IAPL House, 19 Pusa Road, Karol Bagh, New Delhi - 110005 | Patna - 2nd floor, AG Palace, E Boring Canal
Road, Patna, Bihar 800001 | Hyderabad - 1st & 2nd Floor, SM Plaza, RTC X Rd, Indira Park Road, Jawahar Nagar, Hyderabad, Telangana 500020
9821711605 | https://academy.forumias.com | admissions@forumias.academy | helpdesk@forumias.academy
Page 2

SFG 2023 |LEVEL 1 |Test 27 |

Q.1) उत्तरी गोलार्ध की तु लना में दक्षिणी गोलार्ध में पछु आ 3. मानव बखियोों का पवध तीय घाक्षटयोों से पवध त क्षििरोों पर
पवनें अक्षर्क मजबू त और सतत होती हैं । क्ोों? स्थानान्तरण।
1. दक्षिणी गोलार्द्ध में उत्तरी गोलार्द्ध की तु लना में कम 4. घाटी िे त्ोों में फलती-फूलती वनस्पक्षत।
भू भाग है । ऊपर क्षदए गए क्षवकल्पोों में से कौन से सही हैं ?
2. उत्तरी गोलार्द्ध की तु लना में दक्षिणी गोलार्द्ध में a) केवल 1, 2 और 3
कोररओक्षलस बल अक्षर्क होता है। b) केवल 1 और 4
ऊपर क्षदए गए कथनोों में से कौन सा/से सही है /हैं ? c) केवल 2 और 3
a) केवल 1 d) 1, 2, 3 और 4
b) केवल 2
c) 1 और 2 दोनोों Q.5) कालानमक चावल के सों दभध में, क्षनम्नक्षलखित कथनोों
d) न तो 1 और न ही 2 पर क्षवचार करें :
1. चावल की यह क्षकस्म हाल ही में भारतीय क्षवज्ञान
Q.2) वायु राक्षियोों के सों बोंर् में, क्षनम्नक्षलखित कथनोों पर सों स्थान द्वारा क्षवकक्षसत की गई है।
क्षवचार करें : 2. यह अपनी लोंबी दाना के कारण अक्षर्क उपज दे ने
1. वे वायु मोंडलीय हवा के बहुत छोटे पैकेट होते हैं जो वाली चावल की क्षकस्म है ।
तापमान और आद्रता में बडे बदलाव क्षदिाते हैं । 3. यह आमतौर पर उत्तर प्रदे ि के तराई िे त् में उगाया
2. वे तब बनते हैं जब वायु की गक्षत पयाध प्त रूप से कम जाता है ।
होती है । ऊपर क्षदए गए कथनोों में से कौन सा/से सही है /हैं ?
3. वे महासागरोों के ऊपर नही ों बनते हैं , और केवल भू क्षम a) केवल 1
पर ही बन सकते हैं । b) केवल 3
4. ये प्राय: मौसम की दिाओों में पररवतध न लाते हैं । c) केवल 2 और 3
ऊपर क्षदए गए कथनोों में से कौन से सही हैं ? d) 1, 2 और 3
a) केवल 1, 2 और 4
b) केवल 2 और 4 Q.6) उत्तरी भारत में गमी के मौसम में, 'लू' नामक
c) केवल 1 और 4 स्थानीय हवाएँ चलती दे िी जा सकती हैं । यह हवा मुख्य
d) केवल 2 और 3 रूप से क्षनम्नक्षलखित में से क्षकस प्रक्षक्रया का पररणाम है ?
a) चालन
Q.3) समताप रे िा के सों दभध में, क्षनम्नक्षलखित कथनोों पर b) क्षवक्षकरण
क्षवचार करें : c) अक्षभवहन
1. समान तापमान वाले स्थानोों को जोडने वाली ये d) सों वहन
काल्पक्षनक रे िाएँ हैं ।
2. ये रे िाएँ सामान्यतः दे िाों तर रे िाओों के समानाों तर Q.7) कोररओक्षलस बल के सों दभध में, क्षनम्नक्षलखित कथनोों
होती हैं । पर क्षवचार करें :
3. क्षनकटस्थ समतापी रे िाएँ तापमान में बहुत क्रक्षमक 1. यह पवनोों की क्षदिा में क्षविे पण के क्षलए उत्तरदायी है ।
पररवतध न दिाध ती हैं । 2. यह विु के वे ग की क्षदिा के लोंबवत कायध करता है ।
ऊपर क्षदए गए कथनोों में से कौन-सा/से सही है /हैं ? 3. कोररओक्षलस बल भू मध्य रे िा की तु लना में ध्रु वोों पर
a) केवल 1 और 3 अक्षर्क होता है ।
b) केवल 1 और 2 4. अिाों ि इसकी िखि को प्रभाक्षवत करने वाला
c) केवल 2 और 3 एकमात् कारक है ।
d) केवल 1 ऊपर क्षदए गए कथनोों में से कौन से सही हैं ?
a) केवल 1, 2 और 3
Q.4) तापमान के व्यु त्क्रमण के सों भाक्षवत प्रभावोों के सों दभध b) केवल 2 और 4
में, क्षनम्नक्षलखित पर क्षवचार करें : c) केवल 1 और 3
1. वर्ाध का अवरोर्। d) 1, 2, 3 और 4
2. सक्षदधयोों के महीनोों में िराब वायु गु णवत्ता।

Forum Learning Centre: Delhi - 2nd Floor, IAPL House, 19 Pusa Road, Karol Bagh, New Delhi - 110005 | Patna - 2nd floor, AG Palace, E Boring Canal
Road, Patna, Bihar 800001 | Hyderabad - 1st & 2nd Floor, SM Plaza, RTC X Rd, Indira Park Road, Jawahar Nagar, Hyderabad, Telangana 500020
9821711605 | https://academy.forumias.com | admissions@forumias.academy | helpdesk@forumias.academy
Page 3

SFG 2023 |LEVEL 1 |Test 27 |

Q.8) Which of the following statements is/are Q.11) La Nina is suspected to have caused recent
correct regarding the ‘anti-cyclones’? floods in Australia. How is La Nina different
1. Anti-cyclones are the regions of high from El Nino?
pressure around which air circulates. 1. La Nina is characterized by unusually cold
2. Anti-cyclones always brought heavy rain, ocean temperature in equatorial Indian
thus leading to flood like conditions. Ocean whereas El Nino is characterized by
3. Anti-cyclones are more frequent during the unusually warm ocean temperature in the
summer season. equatorial Pacific Ocean.
Select the correct answer using the code given 2. El Nino has adverse effect on south-west
below: monsoon of India, but La Nina has no effect
a) 1 and 3 only on monsoon climate.
b) 2 and 3 only Which of the statements given above is/are
c) 1 only correct?
d) 1, 2 and 3 a) 1 only
b) 2 only
Q.9) With reference to ‘Spatial Distribution of c) Both 1 and 2
Insolation at the earth’s surface’, consider the d) Neither 1 nor 2
following statements:
1. The subtropical deserts receive maximum Q.12) Which among the following is/are ideal
insolation on Earth. condition(s) for the development of a
2. The amount of insolation received by the thunderstorm in a region?
equator is more as compared to the tropics. 1. Presence of moisture in the air
3. The middle and higher latitudes receive less 2. Rapidly rising air
insolation in the winter season than in the 3. Absence of sunlight
summer. Select the correct answer using the code given
4. At the equinoxes, the solar insolation is zero below:
at the poles. a) 1 and 2 only
Which of the following statements given above b) 1 only
are correct? c) 2 and 3 only
a) 1 and 4 only d) 1, 2 and 3
b) 2 and 3 only
Q.13) In context of Climatology, which of the
c) 1, 3 and 4 only
following conditions are necessary for the
d) 3 and 4 only
occurrence of tropical cyclones?
1. Temperature higher than 27° C.
Q.10) Consider the following tribal
2. Presence of the Coriolis force.
communities and the region/states in which
3. Absence of vertical wind
they are found:
4. Upper air convergence above the sea level
Tribal Region/states in
Select the correct answer using the code given
communities which they are
below:
found
a) 1 and 2 only
1. Jarawas Andaman Island
b) 1, 2 and 3 only
2. Dorla Nilgiris hills
c) 1, 2 and 4 only
3. Betta kuruba Bastar region
d) 1, 3 and 4 only
3. Hatti Himachal Pradesh
How many pairs given above are correct? Q.14) In tropical regions particularly in
a) one pair only northern India during summer season local
b) two pairs only winds called ‘loo’ is mainly the outcome of
c) three pairs only which one of the following processes?
d) all four pairs a) Conduction
b) Radiation
c) Advection
d) Convection

Forum Learning Centre: Delhi - 2nd Floor, IAPL House, 19 Pusa Road, Karol Bagh, New Delhi - 110005 | Patna - 2nd floor, AG Palace, E Boring Canal
Road, Patna, Bihar 800001 | Hyderabad - 1st & 2nd Floor, SM Plaza, RTC X Rd, Indira Park Road, Jawahar Nagar, Hyderabad, Telangana 500020
9821711605 | https://academy.forumias.com | admissions@forumias.academy | helpdesk@forumias.academy
Page 4

SFG 2023 |LEVEL 1 |Test 27 |

Q.8) 'प्रक्षत-चक्रवातोों' के सों बोंर् में क्षनम्नक्षलखित में से कौन 1. ला नीना भू मध्यरे िीय क्षहोंद महासागर में असामान्य
सा/से कथन सही है /हैं ? रूप से ठों डे समुद्र के तापमान की क्षविेर्ता है जबक्षक
1. प्रक्षतचक्रवात उच्च दाब के िे त् होते हैं क्षजनके चारोों अल नीनो भू मध्यरे िीय प्रिाों त महासागर में असामान्य
ओर वायु पररचाक्षलत होती है । रूप से गमध समुद्र के तापमान की क्षविेर्ता है ।
2. प्रक्षतचक्रवात हमेिा भारी बाररि लाते हैं , क्षजससे बाढ़ 2. अल नीनो का भारत के दक्षिण-पक्षिम मानसू न पर
जैसी खस्थक्षत पैदा होती है । प्रक्षतकूल प्रभाव पडता है , लेक्षकन ला नीना का मानसू नी
3. प्रक्षतचक्रवात ग्रीष्म ऋतु में अक्षर्क होते हैं । जलवायु पर कोई प्रभाव नही ों पडता है ।
नीचे क्षदए गए कूट का प्रयोग कर सही उत्तर चुक्षनए: ऊपर क्षदए गए कथनोों में से कौन सा/से सही है /हैं ?
a) केवल 1 और 3 a) केवल 1
b) केवल 2 और 3 b) केवल 2
c) केवल 1 c) 1 और 2 दोनोों
d) 1, 2 और 3 d) न तो 1 और न ही 2

Q.9) 'पृथ्वी की सतह पर सू याध तप का स्थाक्षनक क्षवतरण' Q.12) क्षनम्नक्षलखित में से कौन सी एक िे त् में तक्षडत
के सों दभध में, क्षनम्नक्षलखित कथनोों पर क्षवचार करें : झों झावात के क्षवकास के क्षलए आदिध खस्थक्षत है /हैं ?
1. उपोष्णकक्षटबों र्ीय रे क्षगिान पृथ्वी पर अक्षर्कतम 1. वायु में आद्रता की उपखस्थक्षत
सू याध तप प्राप्त करते हैं । 2. ते जी से उठती हुई हवा
2. क्षवर्ुवत रे िा द्वारा प्राप्त सू याध तप की मात्ा उष्ण 3. सू यध के प्रकाि की अनुपखस्थक्षत
कक्षटबों र् की तु लना में अक्षर्क होती है । नीचे क्षदए गए कूट का प्रयोग कर सही उत्तर चुक्षनए:
3. मध्य और उच्च अिाों िोों पर सक्षदधयोों के मौसम में a) केवल 1 और 2
गक्षमधयोों की तु लना में कम सू याध तप प्राप्त होता है । b) केवल 1
4. क्षवर्ुवोों पर, ध्रु वोों पर सू याध तप िून्य होता है । c) केवल 2 और 3
ऊपर क्षदए गए क्षनम्नक्षलखित में से कौन से कथन सही हैं ? d) 1, 2 और 3
a) केवल 1 और 4
b) केवल 2 और 3 Q.13) जलवायु क्षवज्ञान के सों दभध में, उष्णकक्षटबों र्ीय
c) केवल 1, 3 और 4 चक्रवातोों की घटना के क्षलए क्षनम्नक्षलखित में से कौन सी
d) केवल 3 और 4 खस्थक्षत आवश्यक है ?
1. तापमान 27 क्षडग्री से खियस से अक्षर्क।
Q.10) क्षनम्नक्षलखित जनजातीय समुदायोों और उन 2. कोररओक्षलस बल की उपखस्थक्षत।
िे त्ोों/राज्ोों पर क्षवचार करें क्षजनमें वे पाए जाते हैं : 3. ऊर्ध्ाध र्र पवन की अनुपखस्थक्षत
जनजातीय समु दाय क्षेत्र/राज्य जजनमें वे पाए 4. समुद्र तल से ऊपरी वायु अक्षभसरण
जाते हैं नीचे क्षदए गए कूट का प्रयोग कर सही उत्तर चुक्षनए:
1. जारवा अोंडमान द्वीप a) केवल 1 और 2
2. डोरला नीलक्षगरी की पहाक्षडयाँ b) केवल 1, 2 और 3
3. बे ट्टा कुरुबा बिर िे त् c) केवल 1, 2 और 4
4. हट्टी क्षहमाचल प्रदे ि d) केवल 1, 3 और 4
ऊपर क्षदए गए क्षकतने जोडे सही हैं ?
a) केवल एक जोडी Q.14) उष्णकक्षटबों र्ीय िे त्ोों में क्षविेर् रूप से उत्तरी भारत
b) केवल दो जोडे में गमी के मौसम में स्थानीय हवाएँ क्षजन्हें 'लू ' कहा जाता
c) केवल तीन जोडे है , मुख्य रूप से क्षनम्नक्षलखित में से क्षकस प्रक्षक्रया का
d) सभी चार जोडे पररणाम है ?
a) चालन
Q.11) ला नीना के कारण हाल ही में ऑस्ट्रेक्षलया में बाढ़ b) क्षवक्षकरण
आने का सों देह है । ला नीना अल नीनो से कैसे अलग है ? c) अक्षभवहन
d) सों वहन

Forum Learning Centre: Delhi - 2nd Floor, IAPL House, 19 Pusa Road, Karol Bagh, New Delhi - 110005 | Patna - 2nd floor, AG Palace, E Boring Canal
Road, Patna, Bihar 800001 | Hyderabad - 1st & 2nd Floor, SM Plaza, RTC X Rd, Indira Park Road, Jawahar Nagar, Hyderabad, Telangana 500020
9821711605 | https://academy.forumias.com | admissions@forumias.academy | helpdesk@forumias.academy
Page 5

SFG 2023 |LEVEL 1 |Test 27 |

Q.15) Consider the following traditional crafts Q.18) With reference to Polar Vortex, consider
recently seen in news: the following statements:
Traditional Crafts Description 1. It is a whirling cone of high pressure over
1. Patan Patola It is a tie and dye saree the poles.
from Maharashtra. 2. When the vortex becomes strong, a wave of
2. Mata Ni Pachedi It is handmade Gujarati cold air is always pushed southwards.
textile which is offered 3. The term vortex refers to counterclockwise
in temples. flow of air.
3. Pithora painting A wall mural painting Which of the statements given above is/are
done by the Saura tribe correct?
of Odisha a) 1 and 2 only
4. Kanal Brass set Traditional musical b) 1 and 3 only
instrument c) 3 only
manufactured in d) 2 and 3 only
Himachal Pradesh
How many pairs given above are correct? Q.19) Which of the following statements are
a) One pair only correct about the ‘Chinook Winds’?
b) Two pairs only 1. These are warm and dry winds that usually
c) Three pairs only blow in Winter Season.
d) All four pairs 2. They blow along the eastern slopes of the
Rocky Mountains.
Q.16) With reference to the Earth’s Rossby 3. These winds are useful for the ripening of
Waves, consider the following statements: grapes.
1. The equatorial Rossby waves can result in Select the correct answer using the code given
formation of twin cyclones. below:
2. The Rossby waves transfer heat from the a) 1 and 2 only
tropics toward the poles. b) 2 and 3 only
Which of the statements given above is/are c) 1 and 3 only
correct? d) 1, 2 and 3
a) 1 only
b) 2 only Q.20) A Constitution Bench of the Supreme
c) Both 1 and 2 Court has held that a judgment delivered by a
d) Neither 1 nor 2 larger Bench will prevail irrespective of the
number of Judges constituting the majority. In
Q.17) With reference to Planetary Winds and this context, consider the following statements:
their associated regions, consider the following 1. Bench of equal strength can review or
statements: reconsider the decision of another bench of
1. The Westerlies originate in the sub-tropical same strength.
high-pressure belt and blow towards poles. 2. In case of conflict of decisions between the
2. The Westerlies result in heavy precipitation co–equal benches, it is referred to the Chief
and strong winds at horse latitudes. Justice of India.
3. The Westerlies of the Southern hemisphere 3. Decision of larger bench of the court is
are more robust and constant than the always binding on smaller bench of the same
westerlies of the Northern hemisphere. court.
Which of the statements given above are Which of the statements given above are
correct? correct?
a) 1 and 2 only a) 1 and 2 only
b) 3 only b) 2 and 3 only
c) 1 and 3 only c) 1 and 3 only
d) 1, 2 and 3 d) 1, 2 and 3

Forum Learning Centre: Delhi - 2nd Floor, IAPL House, 19 Pusa Road, Karol Bagh, New Delhi - 110005 | Patna - 2nd floor, AG Palace, E Boring Canal
Road, Patna, Bihar 800001 | Hyderabad - 1st & 2nd Floor, SM Plaza, RTC X Rd, Indira Park Road, Jawahar Nagar, Hyderabad, Telangana 500020
9821711605 | https://academy.forumias.com | admissions@forumias.academy | helpdesk@forumias.academy
Page 6

SFG 2023 |LEVEL 1 |Test 27 |

Q.15) हाल ही में समाचारोों में दे िे गए क्षनम्नक्षलखित Q.18) र्ु वीय चक्रवात के सों दभध में, क्षनम्नक्षलखित कथनोों
पारों पररक क्षिल्पोों पर क्षवचार करें : पर क्षवचार करें :
पारं पररक जिल्प जववरण 1. यह ध्रु वोों के ऊपर उच्च दाब का घूमता हुआ िोंकु है ।
1. पाटन पटोला यह महाराष्ट्र की टाई एों ड डाई 2. जब चक्रवात मजबू त हो जाता है , तो ठों डी वायु की
साडी है । लहर हमेिा दक्षिण की ओर र्केलती है ।
2. माता नी पछे डी यह हिक्षनक्षमधत गु जराती कपडा है 3. चक्रवात िब्द वायु के वामावतध प्रवाह को सों दक्षभधत
जो मोंक्षदरोों में चढ़ाया जाता है । करता है ।
3. क्षपथौरा पेंक्षटोंग ओक्षडिा की सौरा जनजाक्षत द्वारा ऊपर क्षदए गए कथनोों में से कौन सा/से सही है /हैं ?
की गई एक दीवार क्षभक्षत्त क्षचत् a) केवल 1 और 2
4.कनाल ब्रास से ट क्षहमाचल प्रदे ि में क्षनक्षमधत b) केवल 1 और 3
पारों पररक वाद्य योंत् c) केवल 3
ऊपर क्षदए गए क्षकतने जोडे सही हैं ? d) केवल 2 और 3
a) केवल एक जोडी
b) केवल दो जोडे Q.19) 'क्षचनूक पवनें' के बारे में क्षनम्नक्षलखित में से कौन सा
c) केवल तीन जोडे कथन सही है ?
d) सभी चार जोडे 1. ये गमध और िुष्क पवनें होती हैं जो आमतौर पर सक्षदधयोों
के मौसम में चलती हैं ।
Q.16) पृथ्वी की रॉस्बी तरों गोों के सों दभध में, क्षनम्नक्षलखित 2. वे रॉकी पवध त के पूवी ढलानोों के साथ बहती हैं ।
कथनोों पर क्षवचार करें : 3. ये पवनें अोंगूरोों को पकाने के क्षलए उपयोगी होती हैं ।
1. भू मध्यरे िीय रॉस्बी तरों गोों के पररणामस्वरूप जुडवाों नीचे क्षदए गए कूट का प्रयोग कर सही उत्तर चुक्षनए:
चक्रवात बन सकते हैं । a) केवल 1 और 2
2. रॉस्बी तरों गें उष्ण कक्षटबों र् से ध्रु वोों की ओर ऊष्मा b) केवल 2 और 3
स्थानाों तररत करती हैं । c) केवल 1 और 3
ऊपर क्षदए गए कथनोों में से कौन सा/से सही है /हैं ? d) 1, 2 और 3
a) केवल 1
b) केवल 2 Q.20) सवोच्च न्यायालय की एक सों क्षवर्ान पीठ ने माना
c) 1 और 2 दोनोों है क्षक एक बडी बें च द्वारा क्षदया गया क्षनणध य बहुमत का
d) न तो 1 और न ही 2 गठन करने वाले न्यायार्ीिोों की सों ख्या के बावजूद प्रबल
होगा। इस सों दभध में क्षनम्नक्षलखित कथनोों पर क्षवचार कीक्षजएः
Q.17) ग्रहीय पवनें और उनसे जुडे िे त्ोों के सों दभध में, 1. समान िखि वाली पीठ उसी िखि की दू सरी पीठ के
क्षनम्नक्षलखित कथनोों पर क्षवचार करें : क्षनणध य की समीिा या पुनक्षवधचार कर सकती है ।
1. पछु आ पवनें उपोष्णकक्षटबों र्ीय उच्च दाब पेटी से 2. सह-समान पीठोों के बीच क्षनणध योों के क्षवरोर् के मामले
क्षनकलती हैं और ध्रु वोों की ओर चलती हैं । में, इसे भारत के मुख्य न्यायार्ीि के पास भे जा जाता
2. पछु वा पवनें घोडे के अिाों िोों पर भारी वर्ाध और ते ज़ है ।
हवाओों का कारण बनती हैं । 3. न्यायालय की बडी पीठ का क्षनणध य हमेिा उसी
3. दक्षिणी गोलार्द्ध की पछु आ पवनें उत्तरी गोलार्द्ध की न्यायालय की छोटी पीठ पर बाध्यकारी होता है ।
पछु आ पवनें की तु लना में अक्षर्क मजबू त और खस्थर ऊपर क्षदए गए कथनोों में से कौन से सही हैं ?
होती हैं । a) केवल 1 और 2
ऊपर क्षदए गए कथनोों में से कौन से सही हैं ? b) केवल 2 और 3
a) केवल 1 और 2 c) केवल 1 और 3
b) केवल 3 d) 1, 2 और 3
c) केवल 1 और 3
d) 1, 2 और 3

Forum Learning Centre: Delhi - 2nd Floor, IAPL House, 19 Pusa Road, Karol Bagh, New Delhi - 110005 | Patna - 2nd floor, AG Palace, E Boring Canal
Road, Patna, Bihar 800001 | Hyderabad - 1st & 2nd Floor, SM Plaza, RTC X Rd, Indira Park Road, Jawahar Nagar, Hyderabad, Telangana 500020
9821711605 | https://academy.forumias.com | admissions@forumias.academy | helpdesk@forumias.academy
Page 7

SFG 2023 |LEVEL 1 |Test 27 |

Q.21) Which one of the following is the 1. The Sharm El-Sheikh Implementation Plan
characteristic climate of the Tropical Savannah was adopted by the parties.
Region? 2. It was decided to set up a Loss and Damage
a) Rainfall throughout the year fund to provide financial assistance to
b) Rainfall in winter only developing countries.
c) An extremely short dry season 3. A binding commitment was reached to
d) A definite dry and wet season phase down the usage of all fossil fuels.
4. A new 5-year work program to promote
Q.22) Which of the following statements climate technology solutions was launched.
correctly describes the ‘Storm Surge’? Which of the statements given above are
a) It is a rise in sea level that occurs during correct?
tropical cyclones or intense storms. a) 1 and 2 only
b) It is a displacement of large volume of water b) 1, 2 and 4 only
caused by movements in Earth's Tectonic c) 3 and 4 only
plates. d) 1, 2, 3 and 4
c) It is an overflow of excessive water on dry
land caused by large storms. Q.26) “It consists of frozen raindrops and
d) None of the statements (a), (b) or (c) is the refrozen melted snow-water. When a layer of
correct description. air having above- freezing point temperature
overlies a subfreezing layer near the ground,
Q.23) With reference to Fog and Mist, consider precipitation takes place in this form.
the following statements: Raindrops leaving the warmer air, encounter
1. The visibility range in fog is more than the the colder air below and as a result, they solidify
visibility range in mist. and reach the ground as small pellets of ice not
2. Fog usually carries much more moisture as bigger than the raindrops from which they are
compared to the mist. formed.”
3. Mist tend to be much denser than the fog. The above description refers to which of the
Which of the statements given above are following types of precipitation?
incorrect? a) Snowfall
a) 1 only b) Sleet
b) 2 and 3 only c) Hailstones
c) 1 and 3 only d) Ice-Crystals
d) 1, 2 and 3
Q.27) Consider the following pairs:
Q.24) Which one of the following is the cause Clouds Description
for formation of Intertropical Convergence 1. Cumulus White, thin, and detached
Zone? clouds cloud having feathery
a) The convergence of equatorial air mass with appearance.
the tropical and sub-tropical air mass near 2. Stratus Clouds formed due to loss of
equator. clouds heat and covering the large
b) The intense heating of mid-latitude belt in portion of the sky.
summers. 3. Nimbus They are featureless mass of
c) The convergence of northeast and clouds vapour, thick enough to block
southeast trade winds near equator. the rays of sun.
d) The high sea surface temperature and high Which of the following pairs given above are
evaporation rate near equator. correct?
a) 1 and 2 only
Q.25) With reference to the outcome of b) 2 and 3 only
Conference of Parties (COP) 27 of the United c) 1 and 3 only
Nations Framework Convention on Climate d) 1, 2 and 3
Change (UNFCCC), consider the following
statements:

Forum Learning Centre: Delhi - 2nd Floor, IAPL House, 19 Pusa Road, Karol Bagh, New Delhi - 110005 | Patna - 2nd floor, AG Palace, E Boring Canal
Road, Patna, Bihar 800001 | Hyderabad - 1st & 2nd Floor, SM Plaza, RTC X Rd, Indira Park Road, Jawahar Nagar, Hyderabad, Telangana 500020
9821711605 | https://academy.forumias.com | admissions@forumias.academy | helpdesk@forumias.academy
Page 8

SFG 2023 |LEVEL 1 |Test 27 |

Q.21) क्षनम्नक्षलखित में से कौन-सी उष्णकक्षटबों र्ीय सवाना 1. पाक्षटधयोों द्वारा िमध अल-िे ि कायाध न्वयन योजना को
प्रदे ि की जलवायु की मुख्य क्षविेर्ता है ? अपनाया गया था।
a) वर्धभर वर्ाध 2. क्षवकासिील दे िोों को क्षवत्तीय सहायता प्रदान करने के
b) केवल िीतकाल में वर्ाध क्षलए एक हाक्षन और िक्षत कोर् स्थाक्षपत करने का
c) अत्योंत अल्पकाक्षलक िुष्क ऋतु क्षनणध य क्षलया गया।
d) क्षनक्षित िुष्क और आद्रध ऋतु 3. सभी जीवाश्म ईोंर्नोों के उपयोग को कम करने के क्षलए
एक बाध्यकारी प्रक्षतबर्द्ता हाक्षसल की गई।
Q.22) क्षनम्नक्षलखित में से कौन सा कथन 'तू फानी 4. जलवायु प्रौद्योक्षगकी समार्ानोों को बढ़ावा दे ने के क्षलए
महोक्षमध'(स्ट्ॉमध सजध) का सही वणधन करता है ? एक नया 5-वर्ीय कायध कायध क्रम िुरू क्षकया गया।
a) यह समुद्र के िर में वृ खर्द् है जो उष्णकक्षटबों र्ीय ऊपर क्षदए गए कथनोों में से कौन से सही हैं ?
चक्रवातोों या तीव्र तू फानोों के दौरान होती है । a) केवल 1 और 2
b) यह पृथ्वी की टे क्टोक्षनक प्लेटोों में हलचल के कारण b) केवल 1, 2 और 4
बडी मात्ा में पानी का क्षवस्थापन है । c) केवल 3 और 4
c) यह बडे तू फानोों के कारण िुष्क भू क्षम पर अत्यक्षर्क d) 1, 2, 3 और 4
पानी का अक्षतप्रवाह है ।
d) कोई भी कथन (a), (b) या (c) सही क्षववरण नही ों है । Q.26) “इसमें जमी हुई बाररि की बूों दें और क्षफर से जमा
हुआ क्षपघला हुआ बफध का पानी होता है । जब हवा की एक
Q.23) कोहरा एवों कुहासा के सों दभध में, क्षनम्नक्षलखित परत क्षहमाों क क्षबोंदु तापमान से ऊपर होती है , तो यह जमीन
कथनोों पर क्षवचार करें : के पास एक सबफ्रीक्षजोंग परत पर आ जाती है , इस रूप
1. कोहरे में दृश्यता सीमा र्ुों र् में दृश्यता सीमा से अक्षर्क में वर्धण होता है । गमध हवा छोडने वाली बाररि की बूों दें ,
होती है । नीचे की ठों डी हवा का सामना करती हैं और
2. कुहासा की तु लना में कोहरे में आमतौर पर अक्षर्क पररणामस्वरूप, वे जम जाती हैं और बफध के छोटे छरों के
नमी होती है । रूप में जमीन पर पहुों च जाती हैं , जो बाररि की बूों दोों से
3. कोहरे की तु लना में कुहासा अक्षर्क सघन होती है । बडे नही ों होते हैं , क्षजससे वे बनते हैं ।
ऊपर क्षदए गए कथनोों में से कौन से गलत हैं ? उपरोि क्षववरण क्षनम्नक्षलखित में से क्षकस प्रकार के वर्ाध
a) केवल 1 को सों दक्षभधत करता है ?
b) केवल 2 और 3 a) क्षहमपात
c) केवल 1 और 3 b) ओलावृ क्षष्ट्
d) 1, 2 और 3 c) ओले
d) बफध-क्षक्रस्ट्ल
Q.24) क्षनम्नक्षलखित में से कौन सा अोंतर-उष्णकक्षटबों र्ीय
Q.27) क्षनम्नक्षलखित यु ग्ोों पर क्षवचार करें :
िे त् के गठन का कारण है ?
मे घ (बादल) जववरण
a) क्षवर्ुवतीय वायु राक्षि का क्षवर्ुवत रे िा के क्षनकट
1. कपासी मेघ सफेद, पतले और अलग-थलग
उष्णकक्षटबों र्ीय और उपोष्णकक्षटबों र्ीय वायु राक्षि के
बादल पोंि जैसे क्षदिते हैं ।
साथ अक्षभसरण।
2. िरी मेघ गमी की कमी और आकाि के
b) ग्रीष्मकाल में मध्य-अिाों ि पेटी का तीव्र ताप।
बडे क्षहस्से को ढकने के कारण
c) भू मध्य रे िा के पास उत्तर-पूवध और दक्षिण-पूवध
बादल बने।
व्यापाररक हवाओों का अक्षभसरण।
3. वर्ाध मेघ वे वाष्प के क्षनराकार क्षपोंड होते
d) भू मध्य रे िा के पास उच्च समुद्री सतह का तापमान
हैं , जो सू यध की क्षकरणोों को रोकने
और उच्च वाष्पीकरण दर।
के क्षलए पयाध प्त मोटे होते हैं ।
ऊपर क्षदए गए क्षनम्नक्षलखित यु ग्ोों में से कौन-से सही हैं ?
Q.25) जलवायु पररवतध न पर सों युि राष्ट्र फ्रेमवकध
a) केवल 1 और 2
कन्वें िन (UNFCCC) के पाक्षटधयोों के सम्मे लन (COP) 27
b) केवल 2 और 3
के पररणाम के सों दभध में, क्षनम्नक्षलखित कथनोों पर क्षवचार
c) केवल 1 और 3
करें :
d) 1, 2 और 3

Forum Learning Centre: Delhi - 2nd Floor, IAPL House, 19 Pusa Road, Karol Bagh, New Delhi - 110005 | Patna - 2nd floor, AG Palace, E Boring Canal
Road, Patna, Bihar 800001 | Hyderabad - 1st & 2nd Floor, SM Plaza, RTC X Rd, Indira Park Road, Jawahar Nagar, Hyderabad, Telangana 500020
9821711605 | https://academy.forumias.com | admissions@forumias.academy | helpdesk@forumias.academy
Page 9

SFG 2023 |LEVEL 1 |Test 27 |

Q.28) Which of the following is the correct Q.31) Why are dewdrops not formed on a cloudy
description of Koeppen’s “C” type of climate? night?
a) Sun is overhead throughout the year, with a a) Clouds absorb the radiation released from
low annual range of temperature and high the Earth’s surface.
annual rainfall. b) Clouds reflect back the Earth’s radiation.
b) Climate present in mid-latitudes c) The Earth’s surface would have low
characterized by warm summers and mild temperature on cloudy nights.
winters. d) Clouds deflect the blowing wind to ground
c) Climate having low rainfall over the region, level.
insufficient for the growth of plants.
d) Climate having cold winters with a large Q.32) Consider the following regarding the
annual range of temperature. characteristics of Equatorial Regions:
1. High temperature and humidity throughout
Q.29) It is a measure of the actual amount of the year
water vapor in the air, regardless of the air's 2. Prevalence of bacteria and insects
temperature. The higher the amount of water 3. Poor quality of soil
vapor, the higher is its value. It is expressed as 4. Presence of dense Forest
grams of moisture per cubic meter of air 5. Commercial extraction of Timber is difficult
(g/m3). Which of the above given options are correct?
Which of the following options best describe a) 1, 3 and 5 only
the paragraph? b) 1, 2, 3 and 4 only
a) Relative Humidity c) 3, 4, and 5 only
b) Dew Point Temperature d) 1, 2, 3, 4, and 5
c) Specific Humidity
d) Absolute Humidity Q.33) “It is a violently rotating column of air
touching the ground. It is attached to and
Q.30) Recently, the Supreme Court upheld the extends from the base of a thunderstorm. It
validity of the 103rd Constitutional Amendment generally develops extremely rapidly, and also
Act. In this context, consider the following dissipates quickly. It is capable of completely
statements: destroying well-made structures and uprooting
1. This amendment required the ratification by trees. It is also known as Twister.”
at least half of the states. Which of the following atmospheric
2. This act provides 10% reservation for phenomena is being described in the above
Economically Weaker Section (EWS) in both paragraph?
public and private educational institutions. a) Blizzard
3. The Supreme Court upheld the 103rd CAA b) Air Mass
by including the EWS group within the c) Tornadoes
Socially and Educationally Backward d) Downbursts
Community (SEBC).
4. The Supreme Court has removed altogether Q.34) Which of the following winds blow over or
the 50% ceiling imposed on reservations to around the Mediterranean Sea?
backward classes. 1. Sirocco
Which of the statements given above is/are 2. Nor wester
correct? 3. Bora
a) 1 and 3 only 4. Santa Ana
b) 1, 2 and 4 only 5. Mistral
c) 2 only Select the correct answer using the code given
d) 2 and 4 only below:
a) 2 and 4 only
b) 1, and 5 only
c) 1, 3 and 5 only
d) 1, 2, 3, 4 and 5

Forum Learning Centre: Delhi - 2nd Floor, IAPL House, 19 Pusa Road, Karol Bagh, New Delhi - 110005 | Patna - 2nd floor, AG Palace, E Boring Canal
Road, Patna, Bihar 800001 | Hyderabad - 1st & 2nd Floor, SM Plaza, RTC X Rd, Indira Park Road, Jawahar Nagar, Hyderabad, Telangana 500020
9821711605 | https://academy.forumias.com | admissions@forumias.academy | helpdesk@forumias.academy
Page 10

SFG 2023 |LEVEL 1 |Test 27 |

Q.28) क्षनम्नक्षलखित में से कौन सा कोपेन के "C" प्रकार a) बादल पृथ्वी की सतह से क्षनकलने वाले क्षवक्षकरण को
की जलवायु का सही वणध न है ? अविोक्षर्त कर लेते हैं ।
a) कम वाक्षर्धक तापमान और उच्च वाक्षर्धक वर्ाध के साथ b) बादल पृथ्वी के क्षवक्षकरण को वापस परावक्षतधत कर दे ते
सू यध पूरे वर्ध क्षसर के ऊपर रहता है । हैं ।
b) मध्य अिाों िोों में मौजूद जलवायु गमध ग्रीष्मकाल और c) बादल वाली रातोों में पृथ्वी की सतह का तापमान कम
हल्की सक्षदधयोों की क्षविेर्ता है । होगा।
c) िे त् में कम वर्ाध वाली जलवायु , पौर्ोों की वृ खर्द् के क्षलए d) बादल बहती हवा को जमीनी िर पर क्षविे क्षपत करते
अपयाध प्त। हैं ।
d) तापमान की बडी वाक्षर्धक सीमा के साथ ठों डी सक्षदधयाँ
वाली जलवायु । Q.32) भू मध्यरे िीय िे त्ोों की क्षविेर्ताओों के सों बोंर् में
क्षनम्नक्षलखित पर क्षवचार करें :
Q.29) यह हवा के तापमान की परवाह क्षकए क्षबना हवा में 1. वर्धभर उच्च तापमान और आद्रध ता
जल वाष्प की वािक्षवक मात्ा का माप है । जलवाष्प की 2. बै क्टीररया और कीडोों का प्रसार
मात्ा क्षजतनी अक्षर्क होगी, उसका मान उतना ही अक्षर्क 3. क्षमट्टी की िराब गुणवत्ता
होगा। इसे हवा के प्रक्षत घन मीटर आद्रता के ग्राम (g/m3) 4. घने जोंगल की उपखस्थक्षत
के रूप में व्यि क्षकया जाता है । 5. इमारती लकडी का व्यावसाक्षयक क्षनष्कर्धण कक्षठन है
क्षनम्न में से कौन सा क्षवकल्प पररच्छे द का सबसे अच्छा ऊपर क्षदए गए क्षवकल्पोों में से कौन से सही हैं ?
वणध न करता है ? a) केवल 1, 3 और 5
a) सापेि आद्रध ता b) केवल 1, 2, 3 और 4
b) ओस क्षबोंदु तापमान c) केवल 3, 4 और 5
d) 1, 2, 3, 4 और 5
c) क्षवक्षिष्ट् आद्रध ता
d) पूणध आद्रध ता Q.33) “यह हवा का एक क्षवनािकारी रूप से घूमने वाला
िों भ है जो जमीन को छूता है । यह तक्षडत झों झावात के
Q.30) हाल ही में, सवोच्च न्यायालय ने 103वें सों क्षवर्ान आर्ार से जुडा और फैला हुआ है । यह आम तौर पर बहुत
सों िोर्न अक्षर्क्षनयम की वै र्ता को बरकरार रिा। इस ते जी से क्षवकक्षसत होता है , और जल्दी से नष्ट् भी हो जाता
सों दभध में क्षनम्नक्षलखित कथनोों पर क्षवचार कीक्षजएः है । यह अच्छी तरह से बने ढाों चोों को पूरी तरह से नष्ट् करने
1. इस सों िोर्न के क्षलए कम से कम आर्े राज्ोों द्वारा और पेडोों को उिाडने में सिम है । इसे क्षिस्ट्र के नाम से
अनुसमथध न की आवश्यकता थी। भी जाना जाता है ।
2. यह अक्षर्क्षनयम सावध जक्षनक और क्षनजी दोनोों क्षििण उपरोि पैराग्राफ में क्षनम्नक्षलखित में से क्षकस वायु मोंडलीय
सों स्थानोों में आक्षथधक रूप से कमजोर वगध (EWS) के घटना का वणध न क्षकया जा रहा है ?
क्षलए 10% आरिण प्रदान करता है । a) तू फान
3. सवोच्च न्यायालय ने सामाक्षजक और िैक्षिक रूप से b) वायु द्रव्यमान
क्षपछडे समुदाय (SEBC) के भीतर EWS समूह को c) टोरनेडो
िाक्षमल करके 103वें सों क्षवर्ान सों िोर्न अक्षर्क्षनयम d) डाउनबस्ट्ध
को बरकरार रिा।
Q.34) क्षनम्नक्षलखित में से कौन सी हवा भू मध्य सागर के
4. सु प्रीम कोटध ने क्षपछडे वगों के क्षलए आरिण पर लगाई
ऊपर या उसके आसपास चलती है ?
गई 50% की सीमा को पूरी तरह से हटा क्षदया है । 1. क्षसरोको
ऊपर क्षदए गए कथनोों में से कौन सा/से सही है /हैं ? 2. नॉवे स्ट्र
a) केवल 1 और 3 3. बोरा
b) केवल 1, 2 और 4 4. सों त एना
c) केवल 2 5. क्षमस्ट्रल
नीचे क्षदए गए कूट का प्रयोग कर सही उत्तर चुक्षनए:
d) केवल 2 और 4
a) केवल 2 और 4
b) केवल 1, और 5
Q.31) बादलोों वाली रात में ओस की बूों दें क्ोों नही ों बनती
c) केवल 1, 3 और 5
हैं ?
d) 1, 2, 3, 4 और 5

Forum Learning Centre: Delhi - 2nd Floor, IAPL House, 19 Pusa Road, Karol Bagh, New Delhi - 110005 | Patna - 2nd floor, AG Palace, E Boring Canal
Road, Patna, Bihar 800001 | Hyderabad - 1st & 2nd Floor, SM Plaza, RTC X Rd, Indira Park Road, Jawahar Nagar, Hyderabad, Telangana 500020
9821711605 | https://academy.forumias.com | admissions@forumias.academy | helpdesk@forumias.academy
Page 11

SFG 2023 |LEVEL 1 |Test 27 |

Q.35) Recently, the Reserve Bank of India (RBI) Which of the statements given above is/are
had allowed international trade settlements correct?
between India and other countries in Rupees. a) 1 only
Which of the following is the most likely impact b) 2 only
of this measure? c) Both 1 and 2
a) Appreciation of Indian rupee against foreign d) Neither 1 nor 2
currencies.
b) Reduction in demand for Indian rupee in the Q.39) Arrange the following surfaces in
global financial market. increasing order of their albedo:
c) It necessarily leads to the decline of India’s 1. Fresh snow
foreign currency reserves. 2. Sand
d) Indian goods will become more competitive 3. Grass
in global markets. 4. Thin Cloud
Select the correct answer using the code given
Q.36) In this type of climate, rainfall occurs below:
throughout the year. They are the regions of a) 3-4-2-1
frontal cyclonic activity or Temperate b) 3-2-4-1
Cyclones. It is also called as Cool Temperate c) 2-3-4-1
Western Margin region. The climate is ideal for d) 2-3-1-4
maximum comfort and mental alertness.
Q.40) With reference to the Pradhan Mantri
Deciduous forest is the natural vegetation of
Kisan Samridhi Kendra (PMKSK) Scheme,
this area.
consider the following statements:
Which of the options given below best
1. It will help in providing agricultural inputs to
describes the above climatic type?
the farmers.
a) British Type Climate
2. It promotes competition among various
b) China Type Climate.
fertiliser companies by ranking their brands
c) Steppe Type Climate
according to its quality.
d) Siberian Type Climate
3. It envisages the conversion of retail fertiliser
shops into PMKSK.
Q.37) Consider the following pairs:
Which of the statements given above is/are
Type of grassland Region
correct?
1. Puszta Hungary
a) 1 and 3 only
2. Prairies Argentina
b) 2 and 3 only
3. Pampas USA
c) 1 only
4. Canterbury New Zealand
d) 1, 2 and 3
5. Velds South Africa
Which of the above pairs are correctly
Q.41) The annual range of temperature in the
matched?
interior of the continents is high as compared
a) 2 and 3 only
to coastal areas. What is/are the
b) 1, 2 and 4 only
reason/reasons?
c) 1, 4 and 5 only
1. Thermal difference between land and water
d) 3, 4 and 5 only
2. Variation in altitude between continents
and oceans
Q.38) Consider the following statements 3. Presence of strong winds in the interior
regarding grasslands: 4. Heavy rains in the interior as compared to
1. A high annual range of temperature is found coasts
over all the temperate grasslands. Select the correct answer using the codes given
2. Savanna Grasslands are characterized by below.
high diurnal range of temperature. a) 1 only
b) 1 and 2 only
c) 2 and 3 only
d) 1, 2, 3 and 4

Forum Learning Centre: Delhi - 2nd Floor, IAPL House, 19 Pusa Road, Karol Bagh, New Delhi - 110005 | Patna - 2nd floor, AG Palace, E Boring Canal
Road, Patna, Bihar 800001 | Hyderabad - 1st & 2nd Floor, SM Plaza, RTC X Rd, Indira Park Road, Jawahar Nagar, Hyderabad, Telangana 500020
9821711605 | https://academy.forumias.com | admissions@forumias.academy | helpdesk@forumias.academy
Page 12

SFG 2023 |LEVEL 1 |Test 27 |

Q.35) हाल ही में, भारतीय ररजवध बैं क (RBI) ने भारत और ऊपर क्षदए गए कथनोों में से कौन सा/से सही है /हैं ?
अन्य दे िोों के बीच रुपये में अोंतराध ष्ट्रीय व्यापार क्षनपटान की a) केवल 1
अनुमक्षत दी थी। क्षनम्न में से कौन सा इस उपाय का सबसे b) केवल 2
सों भाक्षवत प्रभाव है ? c) 1 और 2 दोनोों
a) क्षवदे िी मुद्राओों के मुकाबले भारतीय रुपये की d) न तो 1 और न ही 2
मूल्यवर्ध न।
b) वै क्षिक क्षवत्तीय बाजार में भारतीय रुपये की माों ग में Q.39) क्षनम्नक्षलखित सतहोों को उनके अल्बेडो के बढ़ते
कमी। क्रम में व्यवखस्थत करें :

c) यह आवश्यक रूप से भारत के क्षवदे िी मुद्रा भों डार में 1. ताजा क्षहमपात
क्षगरावट की ओर ले जाता है । 2. रे त
d) भारतीय सामान वैक्षिक बाजारोों में और अक्षर्क 3. घास
प्रक्षतस्पर्ी हो जाएों गे । 4. पतला बादल
नीचे क्षदए गए कूट का प्रयोग कर सही उत्तर चुक्षनए:

Q.36) इस प्रकार की जलवायु में वर्ध भर वर्ाध होती है । वे a) 3-4-2-1

अग्र चक्रवाती गक्षतक्षवक्षर् या समिीतोष्ण चक्रवात के िे त् b) 3-2-4-1

हैं । इसे िीत िीतोष्ण पक्षिमी सीमाों त िे त् भी कहा जाता c) 2-3-4-1


है । जलवायु अक्षर्कतम आराम और मानक्षसक सतकधता के d) 2-3-1-4

क्षलए आदिध है । पणध पाती वन इस िे त् की प्राकृक्षतक


Q.40) प्रर्ानमोंत्ी क्षकसान समृखर्द् केंद्र (PMKS) योजना
वनस्पक्षत है ।
के सों दभध में, क्षनम्नक्षलखित कथनोों पर क्षवचार करें :
नीचे क्षदए गए क्षवकल्पोों में से कौन सा उपरोि जलवायु
1. यह क्षकसानोों को कृक्षर् आदान प्रदान करने में मदद
प्रकार का सबसे अच्छा वणध न करता है ?
करे गा।
a) क्षब्रक्षटि प्रकार की जलवायु
2. यह क्षवक्षभन्न उवध रक कोंपक्षनयोों के बीच उनकी गु णवत्ता
b) चीन प्रकार की जलवायु
के अनुसार उनके ब्राों डोों की रैं क्षकोंग करके प्रक्षतस्पर्ाध
c) स्ट्े पी प्रकार की जलवायु
को बढ़ावा दे ता है ।
d) साइबे ररयन प्रकार की जलवायु
3. इसमें िुदरा उवध रक दु कानोों को पीएमकेएसके में
बदलने की पररकल्पना की गई है ।
Q.37) क्षनम्नक्षलखित जोक्षडयोों पर क्षवचार करें
ऊपर क्षदए गए कथनोों में से कौन सा/से सही है /हैं ?
घास के मै दान का क्षेत्र
a) केवल 1 और 3
प्रकार
b) केवल 2 और 3
1. पुश्ता हों गरी
c) केवल 1
2. प्रेयरीज अजेंटीना
d) 1, 2 और 3
3. पम्पास अमेरीका
4. कैंटरबरी न्यूज़ीलैं ड Q.41) तटीय िे त्ोों की तु लना में महाद्वीपोों के आों तररक
5. वे ल्ड दक्षिण अफ्रीका भाग में तापमान की वाक्षर्धक सीमा अक्षर्क है । इसके
उपरोि में से कौन से जोडे सही सु मेक्षलत हैं ? कारण क्ा है /हैं ?
a) केवल 2 और 3 1. थल और जल के बीच ऊष्मीय अोंतर
b) केवल 1, 2 और 4 2. महाद्वीपोों और महासागरोों के बीच ऊोंचाई में क्षभन्नता
c) केवल 1, 4 और 5 3. आों तररक भाग में ते ज हवाओों की उपखस्थक्षत
d) केवल 3, 4 और 5 4. तटोों की तु लना में आों तररक भाग में भारी वर्ाध
नीचे क्षदए गए कूट का प्रयोग कर सही उत्तर चुक्षनए।
Q.38) घास के मैदानोों के सों बोंर् में क्षनम्नक्षलखित कथनोों पर
a) केवल 1
क्षवचार करें :
b) केवल 1 और 2
1. सभी समिीतोष्ण घास के मैदानोों में तापमान का एक
c) केवल 2 और 3
उच्च वाक्षर्धक पररसर पाया जाता है ।
d) 1, 2, 3 और 4
2. सवाना घास के मैदानोों में तापमान की उच्च दै क्षनक
सीमा होती है ।

Forum Learning Centre: Delhi - 2nd Floor, IAPL House, 19 Pusa Road, Karol Bagh, New Delhi - 110005 | Patna - 2nd floor, AG Palace, E Boring Canal
Road, Patna, Bihar 800001 | Hyderabad - 1st & 2nd Floor, SM Plaza, RTC X Rd, Indira Park Road, Jawahar Nagar, Hyderabad, Telangana 500020
9821711605 | https://academy.forumias.com | admissions@forumias.academy | helpdesk@forumias.academy
Page 13

SFG 2023 |LEVEL 1 |Test 27 |

Q.42) Which among the following are the likely c) It is an ancient style of hand painting done
reasons for Japan having a well-established on cotton clothes.
fishing industry? d) It is an annual folk ritual practised in some
1. Meeting of warm and cold ocean currents parts of Karnataka and Kerala.
near the coast of Japan
2. The presence of the indented coastline Q.46) With reference to ‘Water Vapour in
along the coasts of Japan. atmosphere’, consider the following
3. Lack of plankton on continental shelves statements:
around the coasts of Japan. 1. It is responsible for about half of Earth’s
Select the correct code from the options given greenhouse effect.
below: 2. The amount of water vapour generally
a) 1 and 3 only decreases from the equator towards the
b) 2 and 3 only poles.
c) 1 and 2 only 3. It redistributes heat energy on the Earth
d) 1, 2 and 3 through latent heat energy exchange.
Which of the statements given above is/are
Q.43) With reference to the Tundra region, correct?
consider the following statements: a) 1 only
1. There is a general absence of tree cover in b) 1 and 2 only
the tundra region. c) 2 and 3 only
2. Extraction and mining activities are totally d) 1, 2 and 3
prohibited in the Tundra region.
3. The Tundra plays a significant part in Q.47) The thickness of the troposphere is
mitigating the negative impacts of carbon greatest at the equator because of-
emission. a) Convergence of northeast and the southeast
Which of the above statement is/are correct? trade winds near the equator.
a) 1 and 2 only b) Earth’s axis making an angle of 66½ with the
b) 3 only plane of its orbit.
c) 1 and 3 only c) Heat being transported to great heights by
d) 1, 2 and 3 strong convectional currents.
d) Bulging of the earth at equator as compare
Q.44) Higher concentration of dust particles is to poles
found in subtropical and temperate regions as
compared to equatorial and polar regions. The Q.48) Consider the following pairs:
most appropriate reason for this is: Atmospheric Layers Features
a) Anti-cyclonic conditions in subtropical 1. Stratosphere Nearly free from
high-pressure belts clouds and weather
b) Presence of belt of strong upper-level wind phenomena
circulation 2. Thermosphere Temperature
c) Presence of drier winds in the region increases with
d) Higher elevation of continental landmass in altitude
the region 3. Mesosphere Aurora borealis is
observed here
Q.45) Which one of the following statements How many pairs given above are correctly
best reflects the definition of the term ‘Bhoota matched?
Kola’, that was recently in news? a) None of the pairs
a) It is an ancient style of brass work prevalent b) Only one pair
in Western India.
c) Only two pairs
b) It is a type of puppetry performed by certain
d) All three pairs
tribal communities.

Forum Learning Centre: Delhi - 2nd Floor, IAPL House, 19 Pusa Road, Karol Bagh, New Delhi - 110005 | Patna - 2nd floor, AG Palace, E Boring Canal
Road, Patna, Bihar 800001 | Hyderabad - 1st & 2nd Floor, SM Plaza, RTC X Rd, Indira Park Road, Jawahar Nagar, Hyderabad, Telangana 500020
9821711605 | https://academy.forumias.com | admissions@forumias.academy | helpdesk@forumias.academy
Page 14

SFG 2023 |LEVEL 1 |Test 27 |

Q.42) क्षनम्नक्षलखित में से कौन से सों भाक्षवत कारण जापान b) यह कुछ आक्षदवासी समुदायोों द्वारा की जाने वाली एक
के मछली पकडने के उद्योग की अच्छी तरह से स्थाक्षपत प्रकार की कठपुतली है ।
होने के कारण हैं ? c) यह सू ती कपडोों पर हाथ से की जाने वाली क्षचत्कारी
1. जापान के तट के पास गमध और ठों डी महासागरीय की एक प्राचीन िैली है ।
र्ाराओों का क्षमलना d) यह कनाध टक और केरल के कुछ क्षहस्सोों में प्रचक्षलत
2. जापान के तटोों के साथ दाों तेदार तटरे िा की एक वाक्षर्धक लोक अनुष्ठान है ।
उपखस्थक्षत।
3. जापान के तटोों के आसपास महाद्वीपीय समतल पर Q.46) 'वायु मोंडल में जल वाष्प' के सों दभध में, क्षनम्नक्षलखित
प्लैंकटन की कमी। कथनोों पर क्षवचार करें :
नीचे क्षदए गए क्षवकल्पोों में से सही कूट का चयन करें : 1. यह पृथ्वी के लगभग आर्े ग्रीनहाउस प्रभाव के क्षलए
a) केवल 1 और 3 क्षजम्मेदार है ।
b) केवल 2 और 3 2. क्षवर्ुवत रे िा से ध्रु वोों की ओर सामान्यतः जलवाष्प की
c) केवल 1 और 2 मात्ा घटती जाती है ।
d) 1, 2 और 3 3. यह गु प्त ऊष्मा ऊजाध क्षवक्षनमय के माध्यम से पृथ्वी पर
ऊष्मा ऊजाध का पुनक्षवधतरण करता है ।
Q.43) टुों डरा िे त् के सों दभध में, क्षनम्नक्षलखित कथनोों पर ऊपर क्षदए गए कथनोों में से कौन सा/से सही है /हैं ?
क्षवचार करें : a) केवल 1
1. टुों डरा िे त् में वृ िोों के आच्छादन का सामान्य अभाव है । b) केवल 1 और 2
2. टुों डरा िे त् में क्षनष्कर्धण और िनन गक्षतक्षवक्षर्याँ पूरी c) केवल 2 और 3
तरह से प्रक्षतबों क्षर्त हैं । d) 1, 2 और 3
3. टुों डरा काबध न उत्सजध न के नकारात्मक प्रभावोों को कम
करने में महत्वपूणध भू क्षमका क्षनभाता है । Q.47) िोभमोंडल की मोटाई भू मध्य रे िा पर सबसे
उपरोि में से कौन सा/से कथन सही है /हैं ? अक्षर्क होती है क्ोोंक्षक-
a) केवल 1 और 2 a) भू मध्य रे िा के पास उत्तर-पूवध और दक्षिण-पूवध
b) केवल 3 व्यापाररक हवाओों का अक्षभसरण।
c) केवल 1 और 3 b) पृथ्वी की र्ु री अपनी किा के तल के साथ 66½ क्षडग्री
d) 1, 2 और 3 का कोण बनाती है ।
c) मजबू त सों वहन र्ाराओों द्वारा ऊष्मा का अक्षर्क ऊोंचाई
Q.44) क्षवर्ुवतीय और ध्रु वीय िे त्ोों की तु लना में तक ले जाना।
उपोष्णकक्षटबों र्ीय और समिीतोष्ण िे त्ोों में र्ू ल के कणोों d) ध्रु वोों की तु लना में भू मध्य रे िा पर पृथ्वी का उभार
की उच्च साों द्रता पाई जाती है । इसका सबसे उपयु ि
कारण है : Q.48) क्षनम्नक्षलखित जोक्षडयोों पर क्षवचार करें :
a) उपोष्णकक्षटबों र्ीय उच्च दबाव बे ल्ट में एों टी- वायुमंडलीय परतें जविेषताएं
साइक्लोक्षनक खस्थक्षतयाों 1. समताप मोंडल बादलोों और मौसम की
b) मजबू त ऊपरी-िर पवन पररसों चरण की बे ल्ट की घटनाओों से लगभग
उपखस्थक्षत मुि
c) िे त् में िुष्क हवाओों की उपखस्थक्षत 2. थमोस्फीयर ऊोंचाई के साथ तापमान
d) िे त् में महाद्वीपीय भू भाग का उच्च उन्नयन बढ़ता है
3. मध्य मोंडल अरोरा बोरे क्षलस यहाँ
Q.45) क्षनम्नक्षलखित में से कौन सा कथन 'भू त कोला' िब्द मनाया जाता है
की पररभार्ा को सबसे अच्छा दिाध ता है , जो हाल ही में ऊपर क्षदए गए क्षकतने यु ग् सही सु मेक्षलत हैं ?
िबरोों में था? a) जोडे में से कोई नही ों
a) यह पक्षिमी भारत में प्रचक्षलत पीतल के काम की एक b) केवल एक जोडी
प्राचीन िैली है । c) केवल दो जोडे
d) तीनोों जोडे

Forum Learning Centre: Delhi - 2nd Floor, IAPL House, 19 Pusa Road, Karol Bagh, New Delhi - 110005 | Patna - 2nd floor, AG Palace, E Boring Canal
Road, Patna, Bihar 800001 | Hyderabad - 1st & 2nd Floor, SM Plaza, RTC X Rd, Indira Park Road, Jawahar Nagar, Hyderabad, Telangana 500020
9821711605 | https://academy.forumias.com | admissions@forumias.academy | helpdesk@forumias.academy
Page 15

SFG 2023 |LEVEL 1 |Test 27 |

Q.49) With reference to ‘Solar Radiation on


Earth’, consider the following statements:
1. The surface of Earth receives most of its
energy in long ultraviolet wavelength.
2. The solar output received at the top of the
atmosphere is constant throughout the
year.
3. The angle of inclination of the rays of sun
influences the amount of insolation received
at different latitudes.
Which of the statements given above is/are
correct?
a) 1 only
b) 2 only
c) 2 and 3 only
d) 3 only

Q.50) With reference to the Law Commission of


India, consider the following statements:
1. It is a non-constitutional and non-statutory
body.
2. Its recommendations are binding on the
Government.
3. It can be chaired only by a retired judge of
the Supreme court.
4. Its origin can be traced back to the pre-
independent India.
Which of the statements given above is/are
correct?
a) 1 only
b) 1, 3 and 4 only
c) 2 and 4 only
d) 1 and 4 only

Forum Learning Centre: Delhi - 2nd Floor, IAPL House, 19 Pusa Road, Karol Bagh, New Delhi - 110005 | Patna - 2nd floor, AG Palace, E Boring Canal
Road, Patna, Bihar 800001 | Hyderabad - 1st & 2nd Floor, SM Plaza, RTC X Rd, Indira Park Road, Jawahar Nagar, Hyderabad, Telangana 500020
9821711605 | https://academy.forumias.com | admissions@forumias.academy | helpdesk@forumias.academy
Page 16

SFG 2023 |LEVEL 1 |Test 27 |

Q.49) 'पृथ्वी पर सौर क्षवक्षकरण' के सों दभध में, क्षनम्नक्षलखित


कथनोों पर क्षवचार करें :
1. पृथ्वी की सतह अपनी अक्षर्काों ि ऊजाध लोंबी पराबैं गनी
तरों ग दै ध्यध में प्राप्त करती है ।
2. वायु मोंडल के िीर्ध पर प्राप्त सौर उत्पादन वर्ध भर
खस्थर रहता है ।
3. सू यध की क्षकरणोों के झुकाव का कोण क्षवक्षभन्न अिाों िोों
पर प्राप्त सू याध तप की मात्ा को प्रभाक्षवत करता है ।
ऊपर क्षदए गए कथनोों में से कौन सा/से सही है /हैं ?
a) केवल 1
b) केवल 2
c) केवल 2 और 3
d) केवल 3

Q.50) भारत के क्षवक्षर् आयोग के सों दभध में, क्षनम्नक्षलखित


कथनोों पर क्षवचार करें :
1. यह एक गै र-सों वैर्ाक्षनक और गै र-साों क्षवक्षर्क क्षनकाय
है ।
2. इसकी क्षसफाररिें सरकार के क्षलए बाध्यकारी हैं ।
3. इसकी अध्यिता सवोच्च न्यायालय के से वाक्षनवृ त्त
न्यायार्ीि ही कर सकते हैं ।
4. इसकी उत्पक्षत्त पूवध-स्वतों त् भारत में दे िी जा सकती
है ।
ऊपर क्षदए गए कथनोों में से कौन सा/से सही है /हैं ?
a) केवल 1
b) केवल 1, 3 और 4
c) केवल 2 और 4
d) केवल 1 और 4

Forum Learning Centre: Delhi - 2nd Floor, IAPL House, 19 Pusa Road, Karol Bagh, New Delhi - 110005 | Patna - 2nd floor, AG Palace, E Boring Canal
Road, Patna, Bihar 800001 | Hyderabad - 1st & 2nd Floor, SM Plaza, RTC X Rd, Indira Park Road, Jawahar Nagar, Hyderabad, Telangana 500020
9821711605 | https://academy.forumias.com | admissions@forumias.academy | helpdesk@forumias.academy
Page 1 of 39

SFG 2023 | LEVEL 1 | Test #27 – Solutions |


Q.1) Westerlies in southern hemisphere are stronger and persistent than in northern hemisphere. Why?
1. Southern hemisphere has less landmass as compared to northern hemisphere.
2. Coriolis force is higher in southern hemisphere as compared to northern hemisphere.
Which of the statements given above is/are correct?
a) 1 only
b) 2 only
c) Both 1 and 2
d) Neither 1 nor 2

Ans) a
Exp) Option a is the correct answer.
The Westerlies are prevailing winds from the west toward the east in the middle latitudes between 30°N
and 60°N, and 30°S and 60° S. They originate from the high-pressure areas in the horse latitudes and tend
towards the poles and steer extra tropical cyclones in this general manner. Westerlies in southern
hemisphere are stronger and persistent than in northern hemisphere.
Statement 1 is correct. Land comprises only 19.1% of Southern Hemisphere, In Northern Hemisphere, the
majority of area is composed of land masses. Less landmass in Southern Hemisphere results in lesser
frictional drag in motion of westerlies, thus resulting in stronger and persistent Westerlies.
Statement 2 is incorrect. Coriolis force is equal in both the hemispheres at their respective latitudes. The
magnitude of the Coriolis force depends on the speed of the object and its latitude. The Coriolis force is
zero at the equator and increases toward the poles.
Source) UPSC CSE 2011

Q.2) With respect to the Air Masses, consider the following statements:
1. They are very small packets of atmospheric air showing great changes in temperature and moisture.
2. They are formed when wind speeds are sufficiently low.
3. They do not form over oceans, and can be formed over lands only.
4. They often bring changes in the weather conditions.
Which of the statements given above are correct?
a) 1, 2 and 4 only
b) 2 and 4 only
c) 1 and 4 only
d) 2 and 3 only

Ans) b
Exp) Option b is the correct answer.
When the air remains over a homogeneous area for a sufficiently long time, it acquires the characteristics
of that area. The air with distinctive characteristics in terms of temperature and humidity is called an air
mass.

Forum Learning Centre: Delhi - 2nd Floor, IAPL House, 19 Pusa Road, Karol Bagh, New Delhi - 110005 | Patna - 2nd floor, AG Palace, E Boring Canal Road,
Patna, Bihar 800001 | Hyderabad - 1st & 2nd Floor, SM Plaza, RTC X Rd, Indira Park Road, Jawahar Nagar, Hyderabad, Telangana 500020
9821711605 | https://academy.forumias.com | admissions@forumias.academy | helpdesk@forumias.academy
Page 2 of 39

SFG 2023 | LEVEL 1 | Test #27 – Solutions |

Statement 1 is incorrect: An Air Mass is a large volume of atmospheric air that has almost homogenous
(not greatly varying) characteristics such as temperature and moisture throughout a significant
horizontal distance. Hence this statement is incorrect.
Also, it must be noted that the homogeneity in characteristics typically extends horizontally and not
vertically.
A mass of air wherein there is a great variation in temperature and moisture is known as a “Front”. It is
typically the narrow region where two different types of air masses meet.
Statement 2 is correct: For the Air Mass to form, a large volume of air needs to lie undisturbed over a
piece of land or water, for long periods of time, so that it can absorb the temperature, etc of the underlying
source region. This is possible only when there is low or no wind to disturb this air mass. Hence this
statement is correct.
Statement 3 is incorrect: Air masses are identified based on whether they form over land or over water.
Maritime air masses form over water and are humid. Continental air masses form over land and are dry.
Statement 4 is correct: An Air Mass often brings changes to the weather in destinations where it ends up.
For example, a tropical continental air mass moving from North Africa to Europe will bring dry and warm
conditions to the otherwise cold climate over there. Similarly, when two different types of air masses, such
as warm and humid and cold and dry meet, it gives rise to a Front where a rapid change in temperature,
humidity, etc gives rise to storm like conditions. Hence this statement is correct.
Knowledge Base:
(a) Air masses can extend thousands of kilometres across the surface of Earth, and can reach from ground
level to the stratosphere—16 kilometres (10 miles) into the atmosphere.
(b) Following types of air masses are recognised: (1) Maritime tropical (mT); (2) Continental tropical (cT); (3)
Maritime polar (mP); (4) Continental polar (cP); (5) Continental arctic (cA). Tropical air masses are warm and
polar air masses are cold.
Read More Details: https://www.geographynotes.com/climatology-2/air-masses-meaning-and-
classification-climatology-geography/2814
Source: https://ncert.nic.in/textbook.php?kegy2=10-16
https://education.nationalgeographic.org/resource/air-mass#:~:text=source%20regions.-
,Low%20wind%20speeds,-let%20air%20remain

Q.3) With reference to Isotherms, consider the following statements:


1. These are imaginary lines connecting the places with same temperatures.
2. These lines are generally parallel to the lines of longitude.
3. Closely spaced isotherms indicate very gradual variation in temperature.

Forum Learning Centre: Delhi - 2nd Floor, IAPL House, 19 Pusa Road, Karol Bagh, New Delhi - 110005 | Patna - 2nd floor, AG Palace, E Boring Canal Road,
Patna, Bihar 800001 | Hyderabad - 1st & 2nd Floor, SM Plaza, RTC X Rd, Indira Park Road, Jawahar Nagar, Hyderabad, Telangana 500020
9821711605 | https://academy.forumias.com | admissions@forumias.academy | helpdesk@forumias.academy
Page 3 of 39

SFG 2023 | LEVEL 1 | Test #27 – Solutions |


Which of the statements given above is/ are correct?
a) 1 and 3 only
b) 1 and 2 only
c) 2 and 3 only
d) 1 only

Ans) d
Exp) Option d is the correct answer.
Isotherms are imaginary lines drawn on maps in order to study the variations and distribution in
temperature over various regions of the earth. Following are a few characteristics of Isotherms.
Statement 1 is correct: Isotherms are imaginary lines on a map connecting points experiencing similar
(Iso) temperature (Therm) at a given point of time. Hence this statement is correct.
These lines help understand the distribution and variation of temperature in a region and make
predictions regarding the weather there.
It is not to be confused with other types of imaginary lines such as those connecting places with similar
atmospheric pressure (Isobars), sea water salinity (Isohaline), etc.
Statement 2 is incorrect: These lines are generally parallel to lines of Latitude (not Longitude) in both
Hemispheres. Hence this statement is incorrect.
Latitudes determine the angle at which sun rays fall on a particular place on earth. Spots on Higher
latitudes receive more slanted and hence less intense sun rays, and hence are cooler, and vice versa Hence
Latitude are closely correlated with the distribution of temperature on earth, and hence by extension to
Isotherms as they are the graphical representation of that temperature distribution. This is why Isotherms
are generally parallel to lines of Latitude
Statement 3 is incorrect: Closely spaced isotherms indicate a High Range (i.e., very steep rate of change,
not gradual change) of temperature. Hence this statement is incorrect.
If two isotherms are spaced closely, this means that two places, very close by to each other in distance,
have different types of temperatures. This means that the temperature has changed fast within a very
short distance. Hence the rate of change in temperature is steep not gradual (which would be represented
by widely spaced isotherms).

Source: https://ncert.nic.in/textbook.php?kegy2=9-16 Pg 79

Forum Learning Centre: Delhi - 2nd Floor, IAPL House, 19 Pusa Road, Karol Bagh, New Delhi - 110005 | Patna - 2nd floor, AG Palace, E Boring Canal Road,
Patna, Bihar 800001 | Hyderabad - 1st & 2nd Floor, SM Plaza, RTC X Rd, Indira Park Road, Jawahar Nagar, Hyderabad, Telangana 500020
9821711605 | https://academy.forumias.com | admissions@forumias.academy | helpdesk@forumias.academy
Page 4 of 39

SFG 2023 | LEVEL 1 | Test #27 – Solutions |


Q.4) With reference to the likely effects of the Inversion of Temperature, consider the following:
1. Suppression of rainfall.
2. Poor air quality in winter months.
3. Shifting of human settlements to mountain tops from the mountain valleys.
4. Thriving vegetation in the Valley regions.
Which of the options given above are correct?
a) 1, 2 and 3 only
b) 1 and 4 only
c) 2 and 3 only
d) 1, 2, 3 and 4

Ans) a
Exp) Option a is the correct answer.
Temperature decreases with increasing altitudes in the troposphere at an average rate of 6.5°C per 1000
metres (normal lapse rate) but sometimes this normal trend of decrease of tempera-ture with increasing
heights is reversed under special circumstances i.e., temperature increases upward to a few kilometres
from the earth’s surface. This is called negative lapse rate. Thus, the warm air layer lies over the cold air
layer. This phenomenon meteorologically is called inversion of temperature.
Statement 1 is correct: Having colder, sinking air below the lighter, warmer, more mobile air, results in
very stable conditions where the cold layer acts like a lid. This lid-like layer prevents any convective
movements of particles below it, effectively trapping those things near the earth surface. Thus, the normal
process of convection (hot air rising to higher colder altitudes) is unable to occur. As the warm moisture
bearing air is unable to rise to cooler upper layers and cool adiabatically to create condensation, rain
bearing clouds (cumulonimbus and nimbus) are unable to form. This leads to anticyclonic conditions which
are characterised by aridity and lack of rainfall. Hence this statement is correct.
Statement 2 is correct: As explained above, the inversion creates a lid-like condition near the earth’s
surface. This results in creation of fog (as the cold aids condensation of tiny suspended water droplets),
while preventing pollutants emitted due to human activity during the day from escaping through
convection currents at night. This fog and pollutants combine to form a smog that results in poor air
quality, as is being seen in North India currently, especially in Delhi. This phenomenon is seen mostly in
winters as Surface Inversion occurs only during long winter nights with no winds, as the earth heat gets
radiated during the long winter night and the inverted layer is not disturbed in the absence of winds. Hence
this statement is correct.

Statement 3 is correct and 4 is incorrect: In the Valley Type of Temperature Inversion, during the long,
cold winter nights, the earth’s heat quickly escapes to higher altitudes (nearer the mountain tops), while
its place is occupied by the cold air at the mountain tops which being colder is heavier and hence sinks to
the valley bottom. Thus, valleys are colder than the mountain tops. Also, the valleys experience frost in

Forum Learning Centre: Delhi - 2nd Floor, IAPL House, 19 Pusa Road, Karol Bagh, New Delhi - 110005 | Patna - 2nd floor, AG Palace, E Boring Canal Road,
Patna, Bihar 800001 | Hyderabad - 1st & 2nd Floor, SM Plaza, RTC X Rd, Indira Park Road, Jawahar Nagar, Hyderabad, Telangana 500020
9821711605 | https://academy.forumias.com | admissions@forumias.academy | helpdesk@forumias.academy
Page 5 of 39

SFG 2023 | LEVEL 1 | Test #27 – Solutions |


the morning due to the lid-like effect, and the pollutants are also more heavily concentrated in the valley
region. Thus, the people prefer to build settlements at mountain tops (as there are fewer toxic pollutants).
Hence this statement is correct.
Another effect of this phenomenon is that the vegetation in the valley bottom generally dies (due to the
frost, while that on the mountain top thrives).

Knowledge Base:
Read about the various types of Inversions: https://www.geographynotes.com/climatology-
2/temperature-inversion-meaning-types-and-significance-climatology-geography/2732
Source: https://ncert.nic.in/textbook.php?kegy2=9-16 Pg 81

Q.5) With reference to Kalanamak Rice, consider the following statements:


1. This rice variety has been recently developed by the Indian Institute of Sciences.
2. It is a high yielding rice variety owing to its tall height.
3. It is usually grown in the Terai region of Uttar Pradesh.
Which of the statements given above is/are correct?
a) 1 only
b) 3 only
c) 2 and 3 only
d) 1, 2 and 3

Ans) b
Exp) Option b is the correct answer.
Kalanamak is a rice grown in Nepal and India. It is a black husk, having a strong fragrance.
Statement 1 is incorrect: It is a traditional rice variety has been grown by the people in the Himalayan
terai belt since a long time.
Statement 2 is incorrect: The traditional rice variety has low yield due to its tall height, making it prone
to lodging. Lodging is a condition in which the top of the plant becomes heavy because of grain formation,
the stem becomes weak, and the plant falls on the ground. To address the issue, the Indian Agriculture
Research Institute (IARI) has successfully developed two dwarf varieties of Kalanamak rice. They have
been named Pusa Narendra Kalanamak 1638 and Pusa Narendra Kalanamak 1652.
Statement 3 is correct: The traditional rice variety is grown in 11 districts of the Terai region of north-
eastern Uttar Pradesh and in Nepal.

Forum Learning Centre: Delhi - 2nd Floor, IAPL House, 19 Pusa Road, Karol Bagh, New Delhi - 110005 | Patna - 2nd floor, AG Palace, E Boring Canal Road,
Patna, Bihar 800001 | Hyderabad - 1st & 2nd Floor, SM Plaza, RTC X Rd, Indira Park Road, Jawahar Nagar, Hyderabad, Telangana 500020
9821711605 | https://academy.forumias.com | admissions@forumias.academy | helpdesk@forumias.academy
Page 6 of 39

SFG 2023 | LEVEL 1 | Test #27 – Solutions |


Knowledge Base: The traditional Kalanamak rice has been given the Geographical Indication (GI) tag. It’s
recorded in the GI application that Lord Budhha gifted Kalanamak paddy to the people of Sravasti so that
they remembered him by its fragrance
Source: https://www.thehindu.com/sci-tech/agriculture/fragrant-and-nutritious-kalanamak-rice-
buddhas-gift-to-people-gets-new-powers-and-name/article66069818.ece

Q.6) In Northern India during summer season, local winds called ‘loo’ can be observed to be blown. This
wind is mainly the outcome of which one of the following processes?
a) Conduction
b) Radiation
c) Advection
d) Convection

Ans) c
Exp) Option c is the correct answer.
The Loo is a strong, dusty, gusty, hot and dry summer wind from the west which blows over the Indo-
Gangetic Plain region of North India. It is especially strong in the months of May and June. It is the outcome
of advection process. Advection is a lateral or horizontal transfer of mass, heat, or other property.
Accordingly, winds that blow across Earth's surface represent advectional movements of air.

Advectional Movements of Air


Advective winds move from areas of higher temperature toward areas of lower temperature. In contrast,
convection, the vertical movement of mass or transfer of heat, manifests itself as air currents. Accordingly,
winds are a result of advection, while air currents are a result of convection.
Source: NCERT 11th Geography Unit 4 Climate, Chapter 8 COMPOSITION AND STRUCTURE OF
ATMOSPHERE

Q.7) With reference to the Coriolis Force, consider the following statements:
1. It is responsible for the deflections in the direction of Winds.
2. It acts perpendicular to the direction of velocity of the object.
3. Coriolis Force is higher at the Poles, than at the Equator.
4. The latitude is the sole factor influencing its strength.
Which of the statements given above are correct?
a) 1, 2 and 3 only
b) 2 and 4 only
c) 1 and 3 only
d) 1, 2, 3 and 4

Forum Learning Centre: Delhi - 2nd Floor, IAPL House, 19 Pusa Road, Karol Bagh, New Delhi - 110005 | Patna - 2nd floor, AG Palace, E Boring Canal Road,
Patna, Bihar 800001 | Hyderabad - 1st & 2nd Floor, SM Plaza, RTC X Rd, Indira Park Road, Jawahar Nagar, Hyderabad, Telangana 500020
9821711605 | https://academy.forumias.com | admissions@forumias.academy | helpdesk@forumias.academy
Page 7 of 39

SFG 2023 | LEVEL 1 | Test #27 – Solutions |


Ans) a
Exp) Option a is the correct answer.
The Coriolis Force is a pseudo force that causes objects travelling over great distances to deflect from their
original path (to the right in the Northern Hemisphere and to the Left in the Southern Hemisphere). It is a
result of the Rotation of Earth on its Axis.

Statement 1 is correct: Coriolis Force is a geophysical phenomenon responsible for the deflection in the
direction of flow of fluid systems (winds and ocean currents) on earth surface.
Statement 2 is correct: The Coriolis force acts in a direction perpendicular to the angular velocity of the
rotating frame.
Statement 3 is correct: The Coriolis Force is directly proportional to the angle of latitude. As the latitude
at which horizontally and freely moving objects are located increases, the twisting of the underlying
Earth’s surface and its velocity due to the planet’s rotation increases. Hence the Coriolis Force is
maximum at the Poles and almost absent at the Equator.
Statement 4 is incorrect: The Coriolis Force is dependent on following factors
1) The Velocity of the object travelling over great distances across the latitudes on earth relative to the
Velocity of the earth.
2) The Latitude at which the travelling object is originating. (Sine of the angle of the latitude)
3) The mass of the moving body.
Hence this statement is incorrect as Latitude is not the sole factor influencing the strength of the Coriolis
Force.
Source: https://ncert.nic.in/textbook.php?kegy2=10-16 Pg 86, 87
https://www.geographynotes.com/climatology-2/coriolis-force-definition-and-characteristics-
climatologygeography/2791

Q.8) Which of the following statements is/are correct regarding the ‘anti-cyclones’?
1. Anti-cyclones are the regions of high pressure around which air circulates.
2. Anti-cyclones always brought heavy rain, thus leading to flood like conditions.
3. Anti-cyclones are more frequent during the summer season.
Select the correct answer using the code given below:
a) 1 and 3 only
b) 2 and 3 only
c) 1 only
d) 1, 2 and 3

Ans) a
Exp) Option a is the correct answer.

Forum Learning Centre: Delhi - 2nd Floor, IAPL House, 19 Pusa Road, Karol Bagh, New Delhi - 110005 | Patna - 2nd floor, AG Palace, E Boring Canal Road,
Patna, Bihar 800001 | Hyderabad - 1st & 2nd Floor, SM Plaza, RTC X Rd, Indira Park Road, Jawahar Nagar, Hyderabad, Telangana 500020
9821711605 | https://academy.forumias.com | admissions@forumias.academy | helpdesk@forumias.academy
Page 8 of 39

SFG 2023 | LEVEL 1 | Test #27 – Solutions |


An anticyclone is a weather phenomenon defined as a large-scale circulation of winds around a central
region of high atmospheric pressure.
Statement 1 is correct. Anticyclones are regions of relatively high pressure on horizontal surfaces around
which air circulates clockwise in the Northern Hemisphere and counterclockwise in the Southern
Hemisphere.
Statement 2 is incorrect. Anti-cyclones are indicative of dry weather which mostly remain rainless.
Anticyclones are largely rainless. The sky is free of clouds because of the fact that descending air in the
centre of anticyclone is warmed up at dry adiabatic rate due to subsidence. This is why anticyclones are
indicative of dry weather. This does not mean that anticyclones are always rainless. While passing over
oceans sometimes they pick up moisture and yield light rains or drizzles with moderate clouds.
Statement 3 is correct. In summer, the clear settled conditions associated with anticyclones allow the
Sun's light to warm the ground. This is why anticyclones are more frequent in summers. This can bring
long sunny days and warm temperatures. The weather is normally dry, although occasionally, very hot
temperatures can trigger localized thunderstorms.
Source: savinder singh climatology page 233

Q.9) With reference to ‘Spatial Distribution of Insolation at the earth’s surface’, consider the following
statements:
1. The subtropical deserts receive maximum insolation on Earth.
2. The amount of insolation received by the equator is more as compared to the tropics.
3. The middle and higher latitudes receive less insolation in the winter season than in the summer.
4. At the equinoxes, the solar insolation is zero at the poles.
Which of the following statements given above are correct?
a) 1 and 4 only
b) 2 and 3 only
c) 1, 3 and 4 only
d) 3 and 4 only

Ans) c
Exp) Option c is the correct answer.
The amount of insolation received on the earth’s surface is not uniform everywhere. It varies from place to
place and from time to time. The tropical zone receives the maximum annual insolation. It gradually
decreases towards the poles. Insolation is more in summers and less in winters.
Statement 1 is correct: Maximum insolation is received over the subtropical deserts, where the cloudiness
is the least. The cloudless skies allow for maximum insolation in the subtropical deserts as compare to
other cloudy regions that reflects sunlight.
Statement 2 is incorrect: Equator receives comparatively less insolation than the tropics due to presence
of clouds.
Statement 3 is correct: The middle and higher latitudes receive less radiation in the winter season than
in the summer. Seasonal change has impact on insolation in this region, hence, dissimilarities are found in
the amount of solar radiation.
Statement 4 is correct: At the equinoxes, solar insolation is at a maximum at the equator and is zero at the
poles. At the summer solstice of the northern hemisphere, daily insolation reaches a maximum at the North
Pole because of the 24-hour-long solar day. At the winter solstice, the sun does not rise above the horizon
north of about 66.5°, where solar insolation is zero.
Source: NCERT 11th Geography Unit 4 Climate, Chapter 9 SOLAR RADIATION, HEAT BALANCE
AND TEMPERATURE
https://www.uou.ac.in/lecturenotes/science/MSCGE-
19/Insolation,%20Atmospheric%20temperature%20and%20Heat%20Budget%20of%20the%20Earth.pdf

Forum Learning Centre: Delhi - 2nd Floor, IAPL House, 19 Pusa Road, Karol Bagh, New Delhi - 110005 | Patna - 2nd floor, AG Palace, E Boring Canal Road,
Patna, Bihar 800001 | Hyderabad - 1st & 2nd Floor, SM Plaza, RTC X Rd, Indira Park Road, Jawahar Nagar, Hyderabad, Telangana 500020
9821711605 | https://academy.forumias.com | admissions@forumias.academy | helpdesk@forumias.academy
Page 9 of 39

SFG 2023 | LEVEL 1 | Test #27 – Solutions |


Q.10) Consider the following tribal communities and the region/states in which they are found:
Tribal Region/states in
communities which they are found
1. Jarawas Andaman Island
2. Dorla Nilgiris hills
3. Betta kuruba Bastar region
3. Hatti Himachal Pradesh
How many pairs given above are correct?
a) one pair only
b) two pairs only
c) three pairs only
d) all four pairs

Ans) b
Exp) Option b is the correct answer.
Scheduled Tribes (ST) constitute approximately 8.6% of the population of India i.e., around 10.4 crores.
There are over 705 Scheduled tribes notified under article 342 of the Constitution of India. The Article 342
states that the President may specify the tribes or tribal communities which shall be deemed to be
Scheduled Tribes in relation to that State or Union Territory.
Pair 1 is correct: Jarawas are indigenous people of the Andaman Islands in India. They live in parts of South
Andaman and Middle Andaman Islands. The traditional Jarawa hut is called a Chadda. There are about 500
members of the Jarawa tribe.
Pair 2 is incorrect: Dorla also called Dora are a tribal people community found mainly in Bastar area of
central India. They are mainly found in Dantewada and Bijapur districts of present-day Chhattisgarh.
Pair 3 is incorrect: Betta Kuruba tribe lives in the hilly regions of Karnataka and is one of the few
indigenous communities of the Nilgiris.
Pair 4 is correct: The Hatti tribe is found in the Trans-Giri area of Sirmour district in Himachal Pradesh.
The Hatti community got their name from their tradition of selling homegrown vegetables, crops, meat and
wool etc. at small markets called ‘haat’ in towns.
Knowledge Base:
• To showcase the heritage of tribal communities, the Anthropological Survey of India (AnSI) has recreated
the huts of several communities at its different regional centers. These tribal communities included:
1) Jarawa of Andaman Island
2) Shompen of Great Nicobar Island
3) khasis of Meghalaya
4) Dorla of Bastar region
5) Betta kuruba of Karnataka
Source: https://www.thehindu.com/sci-tech/science/anthropological-survey-of-india-builds-tribal-
hut-replicas-to-promote-unique-heritage/article66104425.ece
https://www.thehindu.com/news/national/cabinet-approves-addition-of-four-tribes-in-himachal-
tamil-nadu-and-chhattisgarh-to-st-list/article65890940.ece
https://blog.forumias.com/anthropological-survey-of-india-builds-tribal-hut-replicas-to-promote-
unique-heritage/

Q.11) La Nina is suspected to have caused recent floods in Australia. How is La Nina different from El Nino?
1. La Nina is characterized by unusually cold ocean temperature in equatorial Indian Ocean whereas El Nino
is characterized by unusually warm ocean temperature in the equatorial Pacific Ocean.

Forum Learning Centre: Delhi - 2nd Floor, IAPL House, 19 Pusa Road, Karol Bagh, New Delhi - 110005 | Patna - 2nd floor, AG Palace, E Boring Canal Road,
Patna, Bihar 800001 | Hyderabad - 1st & 2nd Floor, SM Plaza, RTC X Rd, Indira Park Road, Jawahar Nagar, Hyderabad, Telangana 500020
9821711605 | https://academy.forumias.com | admissions@forumias.academy | helpdesk@forumias.academy
Page 10 of 39

SFG 2023 | LEVEL 1 | Test #27 – Solutions |


2. El Nino has adverse effect on south-west monsoon of India, but La Nina has no effect on monsoon
climate.
Which of the statements given above is/are correct?
a) 1 only
b) 2 only
c) Both 1 and 2
d) Neither 1 nor 2

Ans) d
Exp) Option d is the correct answer.
La Nina events represent periods of below-average sea surface temperatures across the east-central
Equatorial Pacific. It is indicated by sea-surface temperature decrease by more than 0.9℉ for at least five
successive three-month seasons.
Statement 1 is incorrect. La Nina is characterized by unusually cold ocean temperatures in the Equatorial
Pacific compared to El Nino, which is characterized by unusually warm ocean temperatures in the
Equatorial Pacific.
Statement 2 is incorrect. La Nina is favorable to Indian monsoon whereas, El Nino has adverse effect on
south-west monsoon. La Nina leads to heavy floods in Australia. Heavy floods often lead to waterlogging
resulting in poor agricultural output. There are increased temperatures in Western Pacific, Indian Ocean
and off the Somalian coast. In the western Pacific, La Nina increases the potential for landfall in those areas
most vulnerable to their effects, and especially into continental Asia and China.
Source) UPSC 2011

Q.12) Which among the following is/are ideal condition(s) for the development of a thunderstorm in a
region?
1. Presence of moisture in the air
2. Rapidly rising air
3. Absence of sunlight
Select the correct answer using the code given below:
a) 1 and 2 only
b) 1 only
c) 2 and 3 only
d) 1, 2 and 3

Ans) a
Exp) Option a is the correct answer.
Thunderstorms form when warm, moist air rises into cold air. The warm air becomes cooler, which causes
moisture (water vapor) to form small water droplets through condensation. The cooled air drops lower in
the atmosphere, warms, and rises again forming a convection cell. All thunderstorms begin with air rising
into the atmosphere to form a convection cell, but the air can be lifted in different ways. There are three
basic ingredients needed for thunderstorm development: moisture, an unstable atmosphere, and some way
to start the atmosphere moving.
Option 1 is correct: Moisture is necessary to produce the thunderstorm clouds and precipitation.
Presence of sufficient moisture in the air can generate thunderstorms at any time of the year, even in
extreme winter.
Option 2 is correct: Atmospheric instability also plays an important role in thunderstorm development.
Rising air is needed to produce clouds, and rapidly rising air is needed to produce thunderstorms. For

Forum Learning Centre: Delhi - 2nd Floor, IAPL House, 19 Pusa Road, Karol Bagh, New Delhi - 110005 | Patna - 2nd floor, AG Palace, E Boring Canal Road,
Patna, Bihar 800001 | Hyderabad - 1st & 2nd Floor, SM Plaza, RTC X Rd, Indira Park Road, Jawahar Nagar, Hyderabad, Telangana 500020
9821711605 | https://academy.forumias.com | admissions@forumias.academy | helpdesk@forumias.academy
Page 11 of 39

SFG 2023 | LEVEL 1 | Test #27 – Solutions |


air to rise rapidly, it must become buoyant compared to the surrounding air. When the atmosphere is
unstable, air near the ground can become buoyant and rise rapidly through the atmosphere.
Option 3 is incorrect: Sunlight contributes to the trigger motion in the atmosphere. Trigger motion is
some sort of boundary such as a front. Once a thunderstorm has developed, it will continue to generate
boundaries that can trigger additional storms. In the summer, thunderstorms typically develop in the
afternoon when the sun heats air near the ground.
Knowledge Base: The amount of moisture in the air and the air temperature determines how often
thunderstorms form at a particular location. Geographic location also plays a role. Formation of cumulus
clouds, which can lead to thunderstorms, often creates conditions for thunder and lightning to form as
electric charges accumulate within the clouds.
Source: https://scied.ucar.edu/learning-zone/storms/thunderstorms
https://www.weather.gov/safety/lightning-thunderstorm-development

Q.13) In context of Climatology, which of the following conditions are necessary for the occurrence of
tropical cyclones?
1. Temperature higher than 27° C.
2. Presence of the Coriolis force.
3. Absence of vertical wind
4. Upper air convergence above the sea level
Select the correct answer using the code given below:
a) 1 and 2 only
b) 1, 2 and 3 only
c) 1, 2 and 4 only
d) 1, 3 and 4 only

Ans) a
Exp) Option a is the correct answer.
Tropical cyclones are low pressure systems that form over warm tropical waters. It is a rapid rotating storm
originating over tropical oceans from where it draws the energy to develop.
The conditions favourable for the formation and intensification of tropical storms are:
• A pre-existing weak low- pressure area or low-level-cyclonic circulation.
• Large sea surface with temperature higher than 27° C. (Hence, option 1 is correct)
• Presence of the Coriolis force. (Hence, option 2 is correct)
• Small variations in the vertical wind speed. (Hence, option 3 is incorrect)
• Upper air divergence above the sea level system. (Hence, option 4 is incorrect)
Knowledge Base: Cyclones are rapid inward air circulation around a low-pressure area. The air circulates
in an anticlockwise direction in the Northern hemisphere and clockwise in the Southern hemisphere.
Source: NCERT Class XI
https://public.wmo.int/en/our-mandate/focus-areas/natural-hazards-and-disaster-risk-
reduction/tropical-cyclones
http://www.bom.gov.au/cyclone/tropical-cyclone-knowledge-centre/understanding/tc-info/

Q.14) In tropical regions particularly in northern India during summer season local winds called ‘loo’ is
mainly the outcome of which one of the following processes?
a) Conduction
b) Radiation
c) Advection
d) Convection

Forum Learning Centre: Delhi - 2nd Floor, IAPL House, 19 Pusa Road, Karol Bagh, New Delhi - 110005 | Patna - 2nd floor, AG Palace, E Boring Canal Road,
Patna, Bihar 800001 | Hyderabad - 1st & 2nd Floor, SM Plaza, RTC X Rd, Indira Park Road, Jawahar Nagar, Hyderabad, Telangana 500020
9821711605 | https://academy.forumias.com | admissions@forumias.academy | helpdesk@forumias.academy
Page 12 of 39

SFG 2023 | LEVEL 1 | Test #27 – Solutions |


Ans) c
Exp) Option c is the correct answer.
The Loo is a strong, dusty, gusty, hot and dry summer wind from the west which blows over the Indo-
Gangetic Plain region of North India. It is especially strong in the months of May and June. It is the outcome
of advection process. Advection is a lateral or horizontal transfer of mass, heat, or other property.
Accordingly, winds that blow across Earth's surface represent advection movements of air.

Advectional Movements of Air


Advective winds move from areas of higher temperature toward areas of lower temperature. In contrast,
convection, the vertical movement of mass or transfer of heat, manifests itself as air currents. Accordingly,
winds are a result of advection, while air currents are a result of convection.
Source: NCERT 11th Geography Unit 4 Climate, Chapter 8 COMPOSITION AND STRUCTURE OF
ATMOSPHERE

Q.15) Consider the following traditional crafts recently seen in news:


Traditional Description
Crafts
1. Patan Patola It is a tie and dye
saree from
Maharashtra.
2. Mata Ni It is handmade
Pachedi Gujarati textile
which is offered in
temples.
3. Pithora A wall mural
painting painting done by
the Saura tribe of
Odisha
4. Kanal Brass Traditional musical
set instrument
manufactured in
Himachal Pradesh
How many pairs given above are correct?
a) One pair only
b) Two pairs only
c) Three pairs only
d) All four pairs

Forum Learning Centre: Delhi - 2nd Floor, IAPL House, 19 Pusa Road, Karol Bagh, New Delhi - 110005 | Patna - 2nd floor, AG Palace, E Boring Canal Road,
Patna, Bihar 800001 | Hyderabad - 1st & 2nd Floor, SM Plaza, RTC X Rd, Indira Park Road, Jawahar Nagar, Hyderabad, Telangana 500020
9821711605 | https://academy.forumias.com | admissions@forumias.academy | helpdesk@forumias.academy
Page 13 of 39

SFG 2023 | LEVEL 1 | Test #27 – Solutions |


Ans) b
Exp) Option b is the correct answer.
India has rich tradition consisting of crafts and paintings done by the people across the country. At the G20
summit, Prime Minister Narendra Modi gifted traditional artworks from Gujarat and Himachal Pradesh to
world leaders.
Pair 1 is incorrect: Patan Patola is a double ikat saree from Gujarat. The ancient art of double ikat or Patola
woven in pure silk dates to the 11th century. It has equal intensity of colours and design on both sides. This
peculiar quality has its origins in an intricate and difficult technique of dyeing or knot dyeing, known as
‘bandhani’. One of the major practitioners of the dwindling art form is the Salvi family from North Gujarat.
Pochampally sarees are silk saree having intricate geometric patterns handwoven and dyed. It is worn by
the women of Pochampally, Telangana region.
Pair 2 is correct: It is a handmade textile of Gujarat state meant to be an offering in the temple shrines
which house the Mother Goddess. It is done by the artisans of Waghari community. Traditionally, four or
five pieces of Mata-ni-Pachedi are erected to form a temporary shrine
Pair 3 is incorrect: Pithora is a tribal folk art from Chhota Udaipur, Gujarat. They are painted on the walls
of the houses to bring prosperity and peace. Depiction of animals are common especially horses.
Saura paintings are a wall mural painting done by the Saura tribe of Odisha.
Pair 4 is correct: Kanal brass set is a musical instrument from the Mandi and Kullu of Himachal Pradesh.
These traditional musical instruments are now increasingly used as décor objects and are manufactured
by skilled metal craftsperson in Mandi & Kullu district of Himachal Pradesh.
Knowledge Base:
At the G20 Summit, PM Modi presented US President Joe Biden with Kangra miniature paintings; agate
bowls from Kutch to the leaders of France, Germany and Singapore.
Source: https://indianexpress.com/article/explained/modi-gifts-italian-pm-a-patan-patola-scarf-
what-the-ancient-gujarat-art-form-is-8272441/
https://www.financialexpress.com/lifestyle/g20-summit-patan-patola-to-kinnauri-shawl-pm-modi-
gifts-paintings-and-artworks-to-world-leaders/2828620/
https://www.latestly.com/socially/india/news/pm-narendra-modi-gifts-kanal-brass-set-from-mandi-
kullu-to-spanish-prime-minister-latest-tweet-by-ani-4466873.html
https://www.thehindu.com/features/magazine/Showcase-Painted-shrines/article13375288.ece

Q.16) With reference to the Earth’s Rossby Waves, consider the following statements:
1. The equatorial Rossby waves can result in formation of twin cyclones.
2. The Rossby waves transfer heat from the tropics toward the poles.
Which of the statements given above is/are correct?
a) 1 only
b) 2 only
c) Both 1 and 2
d) Neither 1 nor 2

Ans) c
Exp) Option c is the correct answer.
Rossby waves, also known as planetary waves, naturally occur in rotating fluids. Within the Earth's ocean
and atmosphere, these waves form as a result of the rotation of the planet.
Statement 1 is correct. The twin tropical cyclones are caused by equatorial Rossby waves. The twin
cyclone is a vortex (circulation with a definable centre) in the northern hemisphere and another in the
southern hemisphere, and each of these is a mirror image of the other. It is formed when vortex in the
north spins counter clockwise and has a positive spin, while the one in the southern hemisphere spins in

Forum Learning Centre: Delhi - 2nd Floor, IAPL House, 19 Pusa Road, Karol Bagh, New Delhi - 110005 | Patna - 2nd floor, AG Palace, E Boring Canal Road,
Patna, Bihar 800001 | Hyderabad - 1st & 2nd Floor, SM Plaza, RTC X Rd, Indira Park Road, Jawahar Nagar, Hyderabad, Telangana 500020
9821711605 | https://academy.forumias.com | admissions@forumias.academy | helpdesk@forumias.academy
Page 14 of 39

SFG 2023 | LEVEL 1 | Test #27 – Solutions |


the clockwise direction and therefore has a negative spin. Both have positive value of the vorticity which
is a measure of the rotation. Hence resulting in formation of twin cyclones.
Statement 2 is correct. Rossby waves formed in atmosphere help transfer heat from the tropics toward
the poles and cold air toward the tropics in an attempt to return atmosphere to balance. This also help in
locating the jet stream (narrow bands of strong wind in the upper levels of the atmosphere) and mark out
the track of surface low pressure systems. The slow motion of these waves even results in fairly long,
persistent weather patterns.
Source: https://www.thehindu.com/sci-tech/science/explained-the-science-behind-twin-
cyclones/article65402306.ece/amp/
https://oceanservice.noaa.gov/facts/rossby-wave.html

Q.17) With reference to Planetary Winds and their associated regions, consider the following statements:
1. The Westerlies originate in the sub-tropical high-pressure belt and blow towards poles.
2. The Westerlies result in heavy precipitation and strong winds at horse latitudes.
3. The Westerlies of the Southern hemisphere are more robust and constant than the westerlies of the
Northern hemisphere.
Which of the statements given above are correct?
a) 1 and 2 only
b) 3 only
c) 1 and 3 only
d) 1, 2 and 3

Ans) c
Exp) Option c is the correct answer.
The planetary winds are permanent winds which blow throughout the year from low latitude to the other
in response to the latitudinal differences in air pressure. The two most significant winds for climate and
human activities are the trade winds and the westerly winds.
Statement 1 is correct. The westerlies blow from 35-40 degree to 60-65 degree North and South latitude.
They originate in the northern parts of sub-tropical high-pressure cells and blow toward the poles.

Statement 2 is incorrect. The horse latitudes are regions located at about 30 degrees north and south of
the equator. These latitudes are characterized by calm winds and little precipitation. In this region of the
subtropics, winds diverge and either flow toward the poles (known as the prevailing westerlies) or toward
the equator (known as the trade winds). These diverging winds are the result of an area of high pressure,
which is characterized by calm winds, sunny skies, and little or no precipitation.

Forum Learning Centre: Delhi - 2nd Floor, IAPL House, 19 Pusa Road, Karol Bagh, New Delhi - 110005 | Patna - 2nd floor, AG Palace, E Boring Canal Road,
Patna, Bihar 800001 | Hyderabad - 1st & 2nd Floor, SM Plaza, RTC X Rd, Indira Park Road, Jawahar Nagar, Hyderabad, Telangana 500020
9821711605 | https://academy.forumias.com | admissions@forumias.academy | helpdesk@forumias.academy
Page 15 of 39

SFG 2023 | LEVEL 1 | Test #27 – Solutions |


Statement 3 is correct. The westerlies of the southern hemisphere are stronger and persistent due to
the vast expanse of water, while those of the northern hemisphere are irregular because of uneven relief
of vast land-masses.
Source: https://egyankosh.ac.in/bitstream/123456789/46759/1/Unit-14.pdf
NCERT-11TH (OLD) (CH-ATMOSPHERIC PRESSURE, WINDS AND AIRMASS)
https://www.climate-policy-watcher.org/global-climate-2/winds-westerlies.html

Q.18) With reference to Polar Vortex, consider the following statements:


1. It is a whirling cone of high pressure over the poles.
2. When the vortex becomes strong, a wave of cold air is always pushed southwards.
3. The term vortex refers to counterclockwise flow of air.
Which of the statements given above is/are correct?
a) 1 and 2 only
b) 1 and 3 only
c) 3 only
d) 2 and 3 only

Ans) c
Exp) Option c is the correct answer.
A circumpolar vortex, or simply polar vortex, is a large region of cold, rotating air that encircles both of
Earth's polar regions.
Statement 1 is incorrect. Polar vortex is described as a whirling cone of low pressure over the poles.
Statement 2 is incorrect. Normally, when the vortex is strong and healthy, it helps to keep a current of air
(the jet stream) travelling around the globe in almost a circular path. This current keeps the cold air up
north and warm air down south. When the vortex becomes weak there is a lack of a strong low-pressure
system, resulting in jet stream losing the hold to keep it in line, and becoming wavy and all of a sudden,
a river of cold air is pushed down south.
Statement 3 is correct. The term vortex refers to counterclockwise flow of air that keeps the colder air
near the poles.
Knowledge Base:

Forum Learning Centre: Delhi - 2nd Floor, IAPL House, 19 Pusa Road, Karol Bagh, New Delhi - 110005 | Patna - 2nd floor, AG Palace, E Boring Canal Road,
Patna, Bihar 800001 | Hyderabad - 1st & 2nd Floor, SM Plaza, RTC X Rd, Indira Park Road, Jawahar Nagar, Hyderabad, Telangana 500020
9821711605 | https://academy.forumias.com | admissions@forumias.academy | helpdesk@forumias.academy
Page 16 of 39

SFG 2023 | LEVEL 1 | Test #27 – Solutions |


Source: https://indianexpress.com/article/explained/whats-causing-extreme-cold-in-us-midwest-
polar-vortex-5563646/
https://www.downtoearth.org.in/news/climate-change/just-what-exactly-is-a-polar-vortex--62648

Q.19) Which of the following statements are correct about the ‘Chinook Winds’?
1. These are warm and dry winds that usually blow in Winter Season.
2. They blow along the eastern slopes of the Rocky Mountains.
3. These winds are useful for the ripening of grapes.
Select the correct answer using the code given below:
a) 1 and 2 only
b) 2 and 3 only
c) 1 and 3 only
d) 1, 2 and 3

Ans) d
Exp) Option d is the correct answer.
Foehn or Chinook is a type of periodic wind that changes direction periodically with change in season.
Statement 1 and 2 are correct. Chinook winds is a hot dry wind that blows in winter on leeward side of
mountain. It blows east of the Rocky Mountains in Canada and the United States. Chinook's literal
meaning is 'snow eater,' as they aid in the melting of snow.

Statement 3 is correct. The temperature of the wind varies between 15°C and 20°C that helps animal
grazing by melting snow and aids to the suitability for the ripening of grapes.
Knowledge Base:
Chinook wind formed by the adiabatic warming of air that has lost most of its moisture on windward slopes
(orographic lift).
Source: NCERT-11TH (OLD) (CH-ATMOSPHERIC PRESSURE, WINDS AND AIRMASSES)
https://worldinmaps.com/weather-and-climate/foehn-winds/

Q.20) A Constitution Bench of the Supreme Court has held that a judgment delivered by a larger Bench will
prevail irrespective of the number of Judges constituting the majority. In this context, consider the
following statements:
1. Bench of equal strength can review or reconsider the decision of another bench of same strength.
2. In case of conflict of decisions between the co–equal benches, it is referred to the Chief Justice of India.
3. Decision of larger bench of the court is always binding on smaller bench of the same court.

Forum Learning Centre: Delhi - 2nd Floor, IAPL House, 19 Pusa Road, Karol Bagh, New Delhi - 110005 | Patna - 2nd floor, AG Palace, E Boring Canal Road,
Patna, Bihar 800001 | Hyderabad - 1st & 2nd Floor, SM Plaza, RTC X Rd, Indira Park Road, Jawahar Nagar, Hyderabad, Telangana 500020
9821711605 | https://academy.forumias.com | admissions@forumias.academy | helpdesk@forumias.academy
Page 17 of 39

SFG 2023 | LEVEL 1 | Test #27 – Solutions |


Which of the statements given above are correct?
a) 1 and 2 only
b) 2 and 3 only
c) 1 and 3 only
d) 1, 2 and 3

Ans) b
Exp) Option b is the correct answer.
Most of the cases before the Supreme Court is heard and decided by a Bench of two judges called as Division
Bench or of three judges called as full Bench.
Statement 1 is incorrect: Bench of equal strength cannot overrule or reconsider a decision of bench of
same strength. At most it can do is to doubt the correctness of it.
Statement 2 is correct: In case of doubt or conflict between decisions of co-equal Benches, it is referred
to the Chief Justice of India. This is where larger Benches are constituted. The larger Benches examine the
question or correctness of the decision and the majority opinion expressed by them becomes the verdict,
which is binding on the lower Benches.
Statement 3 is correct: The decision of a larger Bench is binding on a smaller Bench of the same court.
Similarly, the decision of a superior court is binding on a lower court. This aims at ensuring that there is
stability and consistency in the decisions of the court. This principle stems from the assumption that a
Bench with more strength is more likely to arrive at a correct decision.
Source: https://www.thehindu.com/opinion/op-ed/judicial-comity-over-
arithmetic/article66022585.ece#:~:text=Photo%20Credit%3A%20PTI-
,Changes%20are%20required%20in%20terms%20of%20how%20larger%20Benches%20are,majority%20i
n%20the%20larger%20Bench.

Q.21) Which one of the following is the characteristic climate of the Tropical Savannah Region?
a) Rainfall throughout the year
b) Rainfall in winter only
c) An extremely short dry season
d) A definite dry and wet season

Ans) d
Exp) Option d is the correct answer.
The Savannah or Sudan Climate is a transitional type of climate found between the equatorial forest and
the trade wind hot deserts. It is confined within the tropics and is best developed in the Sudan. The
savannah landscape is typified by tall grass and short trees.
Option d is correct. The tropical savanna climate has alternating dry and wet seasons. The wet summer
season lasts 6 to 8 months and during these days, there is plenty of rainfall. Winter lasts for 4 to 6 months
and there might be no rains in winter this results in frequent forest fires.
Option a is incorrect. Rainfall throughout the year is a characteristic of Equatorial region.
Option b is incorrect. The Mediterranean region falls under the influence of wet westerlies during winter
season and receive rainfall in winters only.
Option c is incorrect. Tropical Savannah region experience dry conditions for a longer duration than wet
conditions.
Source) UPSC 2012

Q.22) Which of the following statements correctly describes the ‘Storm Surge’?
a) It is a rise in sea level that occurs during tropical cyclones or intense storms.
b) It is a displacement of large volume of water caused by movements in Earth's Tectonic plates.

Forum Learning Centre: Delhi - 2nd Floor, IAPL House, 19 Pusa Road, Karol Bagh, New Delhi - 110005 | Patna - 2nd floor, AG Palace, E Boring Canal Road,
Patna, Bihar 800001 | Hyderabad - 1st & 2nd Floor, SM Plaza, RTC X Rd, Indira Park Road, Jawahar Nagar, Hyderabad, Telangana 500020
9821711605 | https://academy.forumias.com | admissions@forumias.academy | helpdesk@forumias.academy
Page 18 of 39

SFG 2023 | LEVEL 1 | Test #27 – Solutions |


c) It is an overflow of excessive water on dry land caused by large storms.
d) None of the statements (a), (b) or (c) is the correct description.

Ans) a
Exp) Option a is the correct answer.
Option a is correct. A storm surge is a rise in sea level that occurs during tropical cyclones, intense storms
also known as typhoons or hurricanes. The storm surge is a result of atmospheric pressure and Coriolis
force that produces strong winds pushing the water into shore, which can lead to flooding. This makes
storm surges very dangerous for coastal regions. Typhoon Haiyan in 2013 is one of the current examples
of a devastating storm surge.
Option b is incorrect. Tsunamis are giant waves that are produced when a large volume of water is
displaced in an ocean or large lake by movements in Earth's outer layer, or crust like an earthquake, volcanic
eruption, underwater landslide or meteorite.
Option c is incorrect. Flooding is the process where the water overflows or soaks land that is normally dry
caused by large storm or tsunami that leads the water of the sea or river to rush inland.
Source: https://education.nationalgeographic.org/resource/storm-surge
https://oceanservice.noaa.gov/facts/stormsurge-stormtide.html

Q.23) With reference to Fog and Mist, consider the following statements:
1. The visibility range in fog is more than the visibility range in mist.
2. Fog usually carries much more moisture as compared to the mist.
3. Mist tend to be much denser than the fog.
Which of the statements given above are incorrect?
a) 1 only
b) 2 and 3 only
c) 1 and 3 only
d) 1, 2 and 3

Ans) d
Exp) Option d is the correct answer.
When the temperature of an air mass containing a large quantity of water vapour falls suddenly,
condensation takes place within itself on fine dust particles. Both fog and mist follow the same process for
condensation and are tiny water droplets that hang in the air on ground level.
Statement 1 is incorrect: Although both fog and mist cause poor visibility but the visibility range in mist
is more than in fog. Fog is when one can see less than 1,000 metres away, and if the visibility is further than
1,000 metres, we call it mist. Fogs are mini clouds in which condensation takes place around nuclei provided
by the dust, smoke, and the salt particles.
Statement 2 is incorrect: Mist contains more moisture than the fog. In mist each nuclei contains a thicker
layer of moisture. Mists are seen in mountain ranges, where warm air rising the slopes meet a cold surface.
Statement 3 is incorrect: Fog are denser than the mist. Both fog and mist follow the same process of
forming water condensation. However, the formation is less coalescing meaning the merger of tiny water
droplets is less in mist. So, the mist is not as dense as fog.
Source: Class-11, Fundamentals of physical geography, chapter :11
https://www.metoffice.gov.uk/weather/learn-about/weather/types-of-weather/fog/difference-mist-
and-fog#:~:text=Fog%20and%20mist%20differ%20by,metres%2C%20we%20call%20it%20mist.

Forum Learning Centre: Delhi - 2nd Floor, IAPL House, 19 Pusa Road, Karol Bagh, New Delhi - 110005 | Patna - 2nd floor, AG Palace, E Boring Canal Road,
Patna, Bihar 800001 | Hyderabad - 1st & 2nd Floor, SM Plaza, RTC X Rd, Indira Park Road, Jawahar Nagar, Hyderabad, Telangana 500020
9821711605 | https://academy.forumias.com | admissions@forumias.academy | helpdesk@forumias.academy
Page 19 of 39

SFG 2023 | LEVEL 1 | Test #27 – Solutions |


Q.24) Which one of the following is the cause for formation of Intertropical Convergence Zone?
a) The convergence of equatorial air mass with the tropical and sub-tropical air mass near equator.
b) The intense heating of mid-latitude belt in summers.
c) The convergence of northeast and southeast trade winds near equator.
d) The high sea surface temperature and high evaporation rate near equator.

Ans) c
Exp) Option c is the correct answer.
The Inter Tropical Convergence Zone (ITCZ) is a low-pressure zone located at the equator where trade
winds from two hemispheres converge. It is a zone where air tends to ascend.
This convergence zone lies more or less parallel to the equator but moves north or south with the apparent
movement of the sun.
Option c is correct. The Intertropical Convergence Zone is formed when the trade winds of the Northern
and Southern Hemispheres come together. In the northern hemisphere the northeast trade winds
converge with southeast winds from the Southern Hemisphere. The point at which the trade winds
converge forces the air up into the atmosphere, forming the ITCZ.
Source: https://earthobservatory.nasa.gov/images/703/the-intertropical-convergence-zone
https://www.weather.gov/jetstream/itcz

Q.25) With reference to the outcome of Conference of Parties (COP) 27 of the United Nations Framework
Convention on Climate Change (UNFCCC), consider the following statements:
1. The Sharm El-Sheikh Implementation Plan was adopted by the parties.
2. It was decided to set up a Loss and Damage fund to provide financial assistance to developing countries.
3. A binding commitment was reached to phase down the usage of all fossil fuels.
4. A new 5-year work program to promote climate technology solutions was launched.
Which of the statements given above are correct?
a) 1 and 2 only
b) 1, 2 and 4 only
c) 3 and 4 only
d) 1, 2, 3 and 4

Ans) b
Exp) Option b is the correct answer.
The United Nations Climate Change Framework Convention (UNFCCC)’s 27th Conference of Parties (COP
27) concluded recently in Sharm el-Sheikh (Egypt). The COP is the apex decision-making body of the
United Nations Climate Change Framework Convention (UNFCCC).
Statement 1 is correct. The Sharm El-Sheikh Implementation Plan was adopted by COP 27. It highlights
that a global transformation to a low-carbon economy is expected to require investments of at least US$
4-6 trillion a year.
Statement 2 is correct: The COP 27 decided to establish a Loss and Damage fund to provide financial
assistance to vulnerable nations stricken by climate disaster. The fund will assist developing countries
that are particularly vulnerable to the adverse effects of climate change. However, there is no agreement
yet on the mechanism of operation of the fund or how finance should be provided and administered.
Statement 3 is incorrect: At COP27, some countries, led by India, wanted to include a commitment to
phase down all fossil fuels. It was a subject of intense debate but was not included in the final resolution.
Statement 4 is correct: COP27 saw the launch of a new 5-year work program to promote climate
technology solutions in developing countries.

Forum Learning Centre: Delhi - 2nd Floor, IAPL House, 19 Pusa Road, Karol Bagh, New Delhi - 110005 | Patna - 2nd floor, AG Palace, E Boring Canal Road,
Patna, Bihar 800001 | Hyderabad - 1st & 2nd Floor, SM Plaza, RTC X Rd, Indira Park Road, Jawahar Nagar, Hyderabad, Telangana 500020
9821711605 | https://academy.forumias.com | admissions@forumias.academy | helpdesk@forumias.academy
Page 20 of 39

SFG 2023 | LEVEL 1 | Test #27 – Solutions |


Source: https://www.theguardian.com/environment/2022/nov/20/cop27-climate-summit-egypt-key-
outcomes
https://unfccc.int/sites/default/files/resource/cop27_auv_2_cover%20decision.pdf (pg no 9)

Q.26) “It consists of frozen raindrops and refrozen melted snow-water. When a layer of air having above-
freezing point temperature overlies a subfreezing layer near the ground, precipitation takes place in this
form. Raindrops leaving the warmer air, encounter the colder air below and as a result, they solidify and
reach the ground as small pellets of ice not bigger than the raindrops from which they are formed.”
The above description refers to which of the following types of precipitation?
a) Snowfall
b) Sleet
c) Hailstones
d) Ice-Crystals

Ans) b
Exp) Option b is the correct answer.
There is a continuous process of condensation occurring in the free air. When the resistance of the air fails
to hold them against the force of gravity, they fall on to the earth’s surface. So, after the condensation of
water vapour, the release of moisture is known as precipitation. This may take place in liquid or solid form.
Option a is incorrect: Precipitation taking place when the temperature is lower than the 00C in the form
of fine flakes of snow is called snowfall. Moisture is released in the form of hexagonal crystals. These
crystals form flakes of snow.
Option b is correct: Sleet is frozen raindrops and refrozen melted snow-water. When a layer of air having
above freezing point temperature overlies a subfreezing layer near the ground, precipitation takes place in
the form of sleet. Raindrops leaving the warmer air, encounter the colder air below. As a result, they solidify
and reach the ground as small pellets of ice not bigger than the raindrops from which they are formed.
Option c is incorrect: Hailstones have several concentric layers of ice one over the other. Drops of rain
after being released by the clouds become solidified into small rounded solid pieces of ice and which reach
the surface of the earth are called hailstones. They are associated with thunderstorms and individual hail
stones are 5 mm or greater in diameter.
Option d is incorrect: Ice-crystals are falling crystals of ice in the form of needles, columns, or plates.
They occur in very cold region. Also called 'diamond dust', ice crystals appear like fog with individual water
particles forming directly as ice.
Source: Class-11, Fundamentals of physical geography, chapter :11

Q.27) Consider the following pairs:


Clouds Description
1. Cumulus White, thin, and detached
clouds cloud having feathery
appearance.
2. Stratus Clouds formed due to loss
clouds of heat and covering the
large portion of the sky.
3. Nimbus They are featureless mass
clouds of vapour, thick enough to
block the rays of sun.
Which of the following pairs given above are correct?
a) 1 and 2 only

Forum Learning Centre: Delhi - 2nd Floor, IAPL House, 19 Pusa Road, Karol Bagh, New Delhi - 110005 | Patna - 2nd floor, AG Palace, E Boring Canal Road,
Patna, Bihar 800001 | Hyderabad - 1st & 2nd Floor, SM Plaza, RTC X Rd, Indira Park Road, Jawahar Nagar, Hyderabad, Telangana 500020
9821711605 | https://academy.forumias.com | admissions@forumias.academy | helpdesk@forumias.academy
Page 21 of 39

SFG 2023 | LEVEL 1 | Test #27 – Solutions |


b) 2 and 3 only
c) 1 and 3 only
d) 1, 2 and 3

Ans) b
Exp) Option b is the correct answer.
Cloud is a mass of small water droplets or tiny crystals of ice formed by the condensation of the water
vapour in free air at considerable height. They take various shapes according to their elevation. According
to their height, expanse, density and transparency or opaqueness clouds are grouped under four types:
(1) cirrus
(2) cumulus
(3) stratus
(4) nimbus
Pair 1 is incorrect: Cumulus clouds look like cotton wool having flat base. They are generally formed at a
height of 4,000 - 7,000 m. They are usually spotted in fair weather. They exist in patches and can be seen
scattered here and there. Cirrus clouds are white, thin, and detached clouds having a feathery
appearance. They are high clouds, formed at altitudes of 8,000 - 12,000m.

Pair 2 is correct: Stratus clouds low level clouds characterised by layering and covering large portions of
the sky. These clouds are generally formed either due to loss of heat or the mixing of air masses with
different temperatures.
Pair 3 is correct: Nimbus clouds are featureless mass of thick vapour. They are extremely dense and
opaque to the rays of the sun. Sometimes, the clouds are so low that they seem to touch the ground. They
are in colour of black or dark grey. They form at middle levels or very near to the surface.
Source: Class-11, Fundamentals of physical geography, chapter :11

Q.28) Which of the following is the correct description of Koeppen’s “C” type of climate?
a) Sun is overhead throughout the year, with a low annual range of temperature and high annual rainfall.
b) Climate present in mid-latitudes characterized by warm summers and mild winters.
c) Climate having low rainfall over the region, insufficient for the growth of plants.
d) Climate having cold winters with a large annual range of temperature.

Ans) b
Exp) Option b is the correct answer.

Forum Learning Centre: Delhi - 2nd Floor, IAPL House, 19 Pusa Road, Karol Bagh, New Delhi - 110005 | Patna - 2nd floor, AG Palace, E Boring Canal Road,
Patna, Bihar 800001 | Hyderabad - 1st & 2nd Floor, SM Plaza, RTC X Rd, Indira Park Road, Jawahar Nagar, Hyderabad, Telangana 500020
9821711605 | https://academy.forumias.com | admissions@forumias.academy | helpdesk@forumias.academy
Page 22 of 39

SFG 2023 | LEVEL 1 | Test #27 – Solutions |


Koeppen’s scheme of classification of climate is an empirical classification based on mean annual and mean
monthly temperature and precipitation data. Koeppen recognised five major climatic groups namely A, B,
C, D and E. Four of them are based on temperature and one on precipitation.
Option a is incorrect: Group A consists of tropical humid climates. It exists between Tropic of Cancer and
Tropic of Capricorn. The sun is present overhead throughout the year and the presence of Inter Tropical
Convergence Zone (ITCZ) make the climate hot and humid. It has low annual range of temperature and
high annual rainfall. The tropical group is divided into three types, namely
(1) Af- Tropical wet climate
(2) Am - Tropical monsoon climate
(3) Aw- Tropical wet and dry climate.
Option b is correct: Warm temperate (mid-latitude) climates constitute as Koeppen’s type C climate. It
extends from 30° - 50° of latitude on the eastern and western margins of continents. These climates
generally have warm summers with mild winters. They are grouped into four types:
(1) Humid subtropical, i.e., dry in winter and hot in summer (Cwa)
(2) Mediterranean (Cs)
(3) Humid subtropical, i.e., no dry season and mild winter (Cfa)
(4) Marine west coast climate (Cfb)
Option c is incorrect: Dry climate is grouped under type B as per Koeppen’s classification. It is
characterised by very low rainfall that is not adequate for the growth of plants. These climates cover a
very large area of the planet extending over large latitudes from 15° - 60° north and south of the equator.
Dry climates are divided into steppe or semi-arid climate (BS) and desert climate (BW).
Option d is incorrect: Cold snow forest climates comes under group D. It is further sub divided into Df-
cold climate with humid winter and Dw- cold climate with dry winter. Climate with humid winter has
winters that are cold and snowy. The frost-free season is short. The annual ranges of temperature are
large. The weather changes are abrupt and short. Towards the pole, winters are more severe.
Knowledge Base:

Source: Class-11, Fundamentals of physical geography, chapter: World climate and climate change.

Q.29) It is a measure of the actual amount of water vapor in the air, regardless of the air's temperature. The
higher the amount of water vapor, the higher is its value. It is expressed as grams of moisture per cubic
meter of air (g/m3).
Which of the following options best describe the paragraph?
a) Relative Humidity
b) Dew Point Temperature
c) Specific Humidity
d) Absolute Humidity

Forum Learning Centre: Delhi - 2nd Floor, IAPL House, 19 Pusa Road, Karol Bagh, New Delhi - 110005 | Patna - 2nd floor, AG Palace, E Boring Canal Road,
Patna, Bihar 800001 | Hyderabad - 1st & 2nd Floor, SM Plaza, RTC X Rd, Indira Park Road, Jawahar Nagar, Hyderabad, Telangana 500020
9821711605 | https://academy.forumias.com | admissions@forumias.academy | helpdesk@forumias.academy
Page 23 of 39

SFG 2023 | LEVEL 1 | Test #27 – Solutions |


Ans) d
Exp) Option d is the correct answer.
Humidity is the amount of water vapour in the air. Atmospheric water vapour regulates air temperature by
absorbing thermal radiation both from the Sun and the Earth. Water vapour is the ultimate source of all
forms of condensation and precipitation.
Option a is incorrect: Relative Humidity is the percentage ratio of actual water vapour contained in the
given sample of air to the maximum quantity of vapour that air sample can hold at that temperature. If
the temperature of sample of air is raised, its capacity to hold water vapour increases and if so, its relative
humidity will decrease. It is often expressed in percentage.
Option b is incorrect: The dew point is the temperature the air needs to be cooled to (at constant
pressure) in order to achieve a relative humidity of 100%. At this point the air cannot hold more water in
the gas form. Dew point temperature is commonly referred to as the dew point is another way of
expressing humidity. It is a temperature to which a sample of air should be lowered in order to saturate it
with respect to its water vapour content.
Option c is incorrect: Specific Humidity is defined as the ratio of mass of water vapour to the mass of
moist air. Absolute and specific humidity are quite similar in concept. The specific humidity does not vary
as the temperature or pressure of a body of air changes, as long as moisture is not added to or taken away
from it.
Option d is correct: Absolute Humidity is the measure of water vapor in the air, regardless of the
temperature. It is expressed as grams of moisture per cubic meter of air (g/m3). The higher the amount
of water vapor, the higher is the absolute humidity.
Source: https://www.britannica.com/science/humidity
https://egyankosh.ac.in/bitstream/123456789/28451/1/Unit-4.pdf
https://www.zehnderamerica.com/absolute-vs-relative-humidity-whats-the-difference/
https://www.weather.gov/arx/why_dewpoint_vs_humidity#:~:text=The%20dew%20point%20is%20t
he,water%20in%20the%20gas%20form.
https://egyankosh.ac.in/bitstream/123456789/28451/1/Unit-4.pdf

Q.30) Recently, the Supreme Court upheld the validity of the 103rd Constitutional Amendment Act. In this
context, consider the following statements:
1. This amendment required the ratification by at least half of the states.
2. This act provides 10% reservation for Economically Weaker Section (EWS) in both public and private
educational institutions.
3. The Supreme Court upheld the 103rd CAA by including the EWS group within the Socially and
Educationally Backward Community (SEBC).
4. The Supreme Court has removed altogether the 50% ceiling imposed on reservations to backward
classes.
Which of the statements given above is/are correct?
a) 1 and 3 only
b) 1, 2 and 4 only
c) 2 only
d) 2 and 4 only

Ans) c
Exp) Option c is the correct answer.
The 103rd Constitutional Amendment Act provided 10% reservation for the Economically Weaker Sections
(EWS) in education and public employment. The Act was challenged before the Supreme Court as violative

Forum Learning Centre: Delhi - 2nd Floor, IAPL House, 19 Pusa Road, Karol Bagh, New Delhi - 110005 | Patna - 2nd floor, AG Palace, E Boring Canal Road,
Patna, Bihar 800001 | Hyderabad - 1st & 2nd Floor, SM Plaza, RTC X Rd, Indira Park Road, Jawahar Nagar, Hyderabad, Telangana 500020
9821711605 | https://academy.forumias.com | admissions@forumias.academy | helpdesk@forumias.academy
Page 24 of 39

SFG 2023 | LEVEL 1 | Test #27 – Solutions |


of the Constitution of India in 2020. The Supreme Court in Janhit Abhiyan v. Union of India case, through
a 3:2 verdict, has upheld its validity.
Statement 1 is incorrect: The 103rd CAA does not require the ratification by at least half the states.
However, it required constitutional amendment by a special majority of the parliament as it required
changes in fundamental rights (Article 15 and 16) of the Indian constitution.
Statement 2 is correct: The 103rd CAA added Article 15 (6) which provides reservations to economically
weaker sections for admission to educational institutions including Public and private educational
institutions. Thus, the act provides 10% reservation for Economically Weaker Section (EWS) in both public
and private educational institutions.
Statement 3 is incorrect: The Supreme court upheld the 103rd Constitutional Amendment Act by observing
EWS as deemed a separate and distinct category from Social and Economically Backward Classes (SEBC)
as provided in Article 15 and 16 of the Indian constitution. The court also observed poverty as an adequate
marker of deprivation that the State can address through reservations.
Statement 4 is incorrect: The Supreme Court held that the 50% ceiling is for socially and educationally
backward classes. The SC held that the 50% ceiling is for backward classes and it “overstretched to the
reservation provided for entirely different class, consisting of the economically weaker sections”. Thus,
it has not changed the 50% limit in case of backward classes.
Source: https://blog.forumias.com/supreme-courts-judgment-on-ews-reservation/

Q.31) Why are dewdrops not formed on a cloudy night?


a) Clouds absorb the radiation released from the Earth’s surface.
b) Clouds reflect back the Earth’s radiation.
c) The Earth’s surface would have low temperature on cloudy nights.
d) Clouds deflect the blowing wind to ground level.

Ans) b
Exp) Option b is the correct answer.
Dew is small drops of water that form on the ground and other surfaces outdoors during the night.
Option b is correct. Dewdrops are formed when the moisture is deposited in the form of water droplets
on cooler surfaces of solid objects such as stones, grass blades and plant leaves. The ideal conditions for
its formation are clear sky, calm air, high relative humidity, and cold and long nights. When the sky is
clear and the trees and plants are cooler at nights, there is more evaporation of water and hence more dew
formation.
For the formation of dew, it is necessary that the dew point is above the freezing point. The air containing
moisture to its full capacity at a given temperature is said to be saturated. The temperature at which
saturation occurs in a given sample of air is known as dew point.
Dew forms when the temperature becomes equal to the dewpoint. This often happens first at ground level
for two reasons. First, longwave emission causes the earth's surface to cool at night. Condensation requires
the temperature to decrease to the dewpoint. Second, the soil is often the moisture source for the dew.
Warm and moist soils will help with the formation of dew as the soil cools overnight. Cloudy skies reflect
back the Earth’s radiation and that prevents earth’s surface to become cool at night.
Source) UPSC 2019

Q.32) Consider the following regarding the characteristics of Equatorial Regions:


1. High temperature and humidity throughout the year
2. Prevalence of bacteria and insects
3. Poor quality of soil

Forum Learning Centre: Delhi - 2nd Floor, IAPL House, 19 Pusa Road, Karol Bagh, New Delhi - 110005 | Patna - 2nd floor, AG Palace, E Boring Canal Road,
Patna, Bihar 800001 | Hyderabad - 1st & 2nd Floor, SM Plaza, RTC X Rd, Indira Park Road, Jawahar Nagar, Hyderabad, Telangana 500020
9821711605 | https://academy.forumias.com | admissions@forumias.academy | helpdesk@forumias.academy
Page 25 of 39

SFG 2023 | LEVEL 1 | Test #27 – Solutions |


4. Presence of dense Forest
5. Commercial extraction of Timber is difficult
Which of the above given options are correct?
a) 1, 3 and 5 only
b) 1, 2, 3 and 4 only
c) 3, 4, and 5 only
d) 1, 2, 3, 4, and 5

Ans) d
Exp) Option d is the correct answer.
The characteristics of Equatorial Regions are as follows.
Option 1 is correct: There is high average temperatures and humidity throughout the year.
Option 2 is correct: Prevalence of Bacteria and Insect Pests: The hot, wet climate which stimulates rapid
plant growth also encourages the spread of insects and pests. Insects and pests not only spread diseases
but are injurious to crops.
Option 3 is correct: Poor quality of soil: The region has poor soil because of the amount of rain in the area.
Option 4 is correct: Presence of Dense Forest: The Forest is so dense that it is quite a problem to clear a
small patch of it and even more difficult to maintain it.
Option 5 is correct: Commercial extraction of Timber is difficult. The trees do not occur in homogenous
stands, there are no frozen surfaces to facilitate logging and the tropical hardwoods are sometimes too
heavy to float in the rivers.
Source: Pg 119, ch15 The Hot, Wet Equatorial climate, G C Leong

Q.33) “It is a violently rotating column of air touching the ground. It is attached to and extends from the
base of a thunderstorm. It generally develops extremely rapidly, and also dissipates quickly. It is capable of
completely destroying well-made structures and uprooting trees. It is also known as Twister.”
Which of the following atmospheric phenomena is being described in the above paragraph?
a) Blizzard
b) Air Mass
c) Tornadoes
d) Downbursts

Ans) c
Exp) Option c is the correct answer.
Tornadoes are violently spinning, funnel-shaped columns of air that extend from the dark
thunderclouds (bottom layer of the sky where they form) to the ground. Tornadoes are also known as
twisters. The wind from a tornado can be more than 250 Km/Hour. Tornadoes occur all over the planet,
but the United States leads the world in the number of storms followed by Argentina and Bangladesh. Some
tornadoes are clearly visible, while rain or nearby low-hanging clouds obscure others. Tornadoes develop
extremely rapidly, and may dissipate just a quickly. Most tornadoes are on the ground for less than 15
minutes.
Source: https://www.weather.gov/safety/tornado

Q.34) Which of the following winds blow over or around the Mediterranean Sea?
1. Sirocco
2. Nor wester
3. Bora
4. Santa Ana

Forum Learning Centre: Delhi - 2nd Floor, IAPL House, 19 Pusa Road, Karol Bagh, New Delhi - 110005 | Patna - 2nd floor, AG Palace, E Boring Canal Road,
Patna, Bihar 800001 | Hyderabad - 1st & 2nd Floor, SM Plaza, RTC X Rd, Indira Park Road, Jawahar Nagar, Hyderabad, Telangana 500020
9821711605 | https://academy.forumias.com | admissions@forumias.academy | helpdesk@forumias.academy
Page 26 of 39

SFG 2023 | LEVEL 1 | Test #27 – Solutions |


5. Mistral
Select the correct answer using the code given below:
a) 2 and 4 only
b) 1, and 5 only
c) 1, 3 and 5 only
d) 1, 2, 3, 4 and 5

Ans) c
Exp) Option c is the correct answer.
Many local winds are common around the Mediterranean Sea due to the varied topography of the region.
The various local wind around the Mediterranean Sea are as follows:
1) Sirocco: This is a hot, dry, dusty wind which originates in the Sahara Desert. It blows outward from the
desert interior to the cooler Mediterranean Sea. It is known by different local names such as Chili in Tunisia,
Ghibli in Libya, Leveche in Spain, and Khamsin in Egypt and Malta. In Adriatic & Aegean Sea, this hot wind
is known as Gharbi. Hence, option 1 is correct.
2) Bora Wind: It is Cold north-easterly wind experienced along the Adriatic. It is caused by a difference in
pressure between continental Europe & Mediterranean. Bora wind is more violent than the Mistral & speeds
of over 100 mph have been recorded. Hence, option 3 is correct.
3) Mistral Wind: The mistral is a strong, cold, northwesterly wind that blows from southern France into
the Gulf of Lion in the northern Mediterranean. Hence, option 5 is correct.
4) Fohn wind: Fohn is experienced in the valleys of northern Alps, particularly in Switzerland in spring.
Option 2 is incorrect: Nor wester/ Kalbaishakhi wind: During the pre-monsoon season, the eastern and
north-eastern state of India experience the appearance of a special type thunderstorm known as nor-
wester.
Option 4 is incorrect: The Santa Ana winds are strong, extremely dry downslope winds that originate inland
and affect coastal California in USA.

Source: Pg 143, Ch 19 The Warm Temperate Western Margin, G C Leong

Q.35) Recently, the Reserve Bank of India (RBI) had allowed international trade settlements between India
and other countries in Rupees. Which of the following is the most likely impact of this measure?
a) Appreciation of Indian rupee against foreign currencies.
b) Reduction in demand for Indian rupee in the global financial market.
c) It necessarily leads to the decline of India’s foreign currency reserves.

Forum Learning Centre: Delhi - 2nd Floor, IAPL House, 19 Pusa Road, Karol Bagh, New Delhi - 110005 | Patna - 2nd floor, AG Palace, E Boring Canal Road,
Patna, Bihar 800001 | Hyderabad - 1st & 2nd Floor, SM Plaza, RTC X Rd, Indira Park Road, Jawahar Nagar, Hyderabad, Telangana 500020
9821711605 | https://academy.forumias.com | admissions@forumias.academy | helpdesk@forumias.academy
Page 27 of 39

SFG 2023 | LEVEL 1 | Test #27 – Solutions |


d) Indian goods will become more competitive in global markets.

Ans) a
Exp) Option a is the correct answer.
Recently, the RBI had issued a circular allowing international trade settlement in Rupees. It came in the
backdrop of continuous fall in the value of Indian Rupee against the US Dollar. It is expected that the
decision to allow Indian traders to settle imports and exports in rupees will ease the demand for US Dollar
and help control the falling exchange rate.
Option a is correct: The trade settlement in Indian rupees will likely result in the increasing demand for
Indian rupee in the global exchange rate market as countries will start converting foreign currency into
rupee to engage in trade transactions with India. This will likely result in appreciation of Indian rupee.
Option b is incorrect: With this measure, the value of Indian rupee against foreign currencies such as US
Dollar, Euro etc., will increase, thus appreciation of Indian rupee. As the demand for the Indian rupee is
rising, its value will start rising.
Option c is incorrect: This measure will not necessarily reduce the foreign currency reserve of the
country. The measure will reduce India’s dependency on dollars and other Hard currencies, which is likely
to reduce the depletion of forex reserves of India.
Option d is incorrect: This measure will likely lead to appreciation of Indian rupee. With the increase in
value of Indian rupee against foreign currency, Indian goods will lose their trade competitiveness because
foreign countries have to pay more to buy Indian goods.
Source: https://indianexpress.com/article/explained/explained-rbi-international-trade-settlements-
rupees-8025178/

Q.36) In this type of climate, rainfall occurs throughout the year. They are the regions of frontal cyclonic
activity or Temperate Cyclones. It is also called as Cool Temperate Western Margin region. The climate is
ideal for maximum comfort and mental alertness. Deciduous forest is the natural vegetation of this area.
Which of the options given below best describes the above climatic type?
a) British Type Climate
b) China Type Climate.
c) Steppe Type Climate
d) Siberian Type Climate

Ans) a
Exp) Option a is the correct answer.
Option a is correct: British Type Climate is also called as Cool Temperate Western Margin Climate. They
are under the influence of the Westerlies all-round the year. They are the regions of frontal cyclonic
activity or Temperate Cyclones. Rainfall occurs throughout the year. The natural vegetation of this
climatic type is deciduous forest. The deciduous trees occur in pure stands and have greater lumbering
value. The climate is ideal for maximum comfort and mental alertness.
Option b is incorrect: Warm Temperate Eastern Margin Climate is also called as China Type Climate.
Characterized by a warm moist summer and a cool, dry winter. There is fairly uniform distribution of rainfall
throughout the year. Local storms like typhoons and hurricanes occur here.
Option c is incorrect: Temperate Continental/Grassland Climate is also called as Steppe Climate. They
lie in the Westerly wind belt, and in interiors of the continents. Grasslands are practically treeless due to
continentality. Grasses are tall, fresh and nutritious and are ideal for extensive wheat cultivation.
Temperatures vary greatly between summer and winter. The steppe type of climate in the southern
hemisphere is never severe.

Forum Learning Centre: Delhi - 2nd Floor, IAPL House, 19 Pusa Road, Karol Bagh, New Delhi - 110005 | Patna - 2nd floor, AG Palace, E Boring Canal Road,
Patna, Bihar 800001 | Hyderabad - 1st & 2nd Floor, SM Plaza, RTC X Rd, Indira Park Road, Jawahar Nagar, Hyderabad, Telangana 500020
9821711605 | https://academy.forumias.com | admissions@forumias.academy | helpdesk@forumias.academy
Page 28 of 39

SFG 2023 | LEVEL 1 | Test #27 – Solutions |


Option d is incorrect: Cool Temperature Continental climate is also called as Siberian Climate. The
Siberian Climate is absent in the southern hemisphere because of the narrowness of the southern
continents. The vegetation is predominantly evergreen coniferous forest. It stretches in a great, continuous
belt across North America, Europe and Asia. The greatest single band of the coniferous forest is the taiga
in Siberia. The climate is characterized by a bitterly cold winter of long duration, and a cool brief summer.

Source: Pg161, Ch 22 The Cool Temperate Western Margin Climate, G C Leong

Q.37) Consider the following pairs:


Type of grassland Region
1. Puszta Hungary
2. Prairies Argentina
3. Pampas USA
4. Canterbury New Zealand
5. Velds South Africa
Which of the above pairs are correctly matched?
a) 2 and 3 only
b) 1, 2 and 4 only
c) 1, 4 and 5 only
d) 3, 4 and 5 only

Ans) c
Exp) Option c is the correct answer.
Steppes are a class of regions characterized by vast, wide-open grasslands. In this area, trees are generally
absent, except near lakes or rivers. The climate of the steppes differs from region to region. Steppe biomes
comprise the montane grasslands and shrub lands biome and the temperate grasslands, savannas, and
shrub lands biome.
Pair 1 is correct: The Hungarian Puszta is a temperate grassland biome of the Alfold or Great Hungarian
Plain. It is an exclave of the Eurasian Steppe and lies around the river Tisza in the eastern part of Hungary,
as well as in the western part of the country and in the Burgenland of Austria.

Forum Learning Centre: Delhi - 2nd Floor, IAPL House, 19 Pusa Road, Karol Bagh, New Delhi - 110005 | Patna - 2nd floor, AG Palace, E Boring Canal Road,
Patna, Bihar 800001 | Hyderabad - 1st & 2nd Floor, SM Plaza, RTC X Rd, Indira Park Road, Jawahar Nagar, Hyderabad, Telangana 500020
9821711605 | https://academy.forumias.com | admissions@forumias.academy | helpdesk@forumias.academy
Page 29 of 39

SFG 2023 | LEVEL 1 | Test #27 – Solutions |


Pair 2 is incorrect: The prairies are temperate grasslands of North America. They are enormous stretches
of flat grassland with moderate temperature, moderate rainfall, and few trees and in the middle of North
America they are usually golden, wheat-covered land.
Pair 3 is incorrect: The Pampas grassland is found in South America. Pampas are the temperate grasslands
of Argentina.
Pair 4 is correct: The Canterbury grasslands is an ecoregion of the South Island, New Zealand, part of the
wider tussock grasslands of New Zealand. This type of Grassland is frequently found between woods and
desert.
Pair 5 is correct: The temperate grasslands of South Africa are called the Velds. The Velds have a mild
climate due to the influence of the Indian Oceans. The vegetation cover is sparse, with red grasses growing
in the bush Velds and acacia and maroola growing in the high Velds.
Knowledge Base:

Source: https://ncert.nic.in/ncerts/l/gess209.pdf
https://ncert.nic.in/textbook/pdf/kegy212.pdf
https://ncert.nic.in/textbook/pdf/gess206.pdf

Q.38) Consider the following statements regarding grasslands:


1. A high annual range of temperature is found over all the temperate grasslands.
2. Savanna Grasslands are characterized by high diurnal range of temperature.
Which of the statements given above is/are correct?
a) 1 only
b) 2 only
c) Both 1 and 2
d) Neither 1 nor 2

Ans) b
Exp) Option b is the correct answer.
Temperate grasslands are found in the regions with temperate and semi-arid to semi-humid climates. The
Veldts of South Africa, the Puszta of Hungary, the Pampas of Argentina and Uruguay, the Steppes, and the
plains and Prairies of Central North America are Temperate Grasslands

Forum Learning Centre: Delhi - 2nd Floor, IAPL House, 19 Pusa Road, Karol Bagh, New Delhi - 110005 | Patna - 2nd floor, AG Palace, E Boring Canal Road,
Patna, Bihar 800001 | Hyderabad - 1st & 2nd Floor, SM Plaza, RTC X Rd, Indira Park Road, Jawahar Nagar, Hyderabad, Telangana 500020
9821711605 | https://academy.forumias.com | admissions@forumias.academy | helpdesk@forumias.academy
Page 30 of 39

SFG 2023 | LEVEL 1 | Test #27 – Solutions |


Statement 1 is incorrect. In the northern hemisphere, the climate of Temperate grasslands is continental
with a high range of annual temperature. Temperate grassland in the northern hemisphere is extensive and
continental.
In the southern hemisphere, the climate is never severe. It is mainly due to the narrowness of the
temperate portions of the southern continents. Here, the annual range of temperature is very low.
Temperate grassland in the southern hemisphere is restricted and less continental i.e., their annual range
of temperature is very low.
Statement 2 is correct. The tropical savanna climate is also called the tropical wet and dry climate. It is the
climate experienced in savanna or tropical grassland regions of the world. These places are located near
the equator, and they lie between the Southern and the Northern Tropics.
Savanna (or Sudan) type of climate has alternate hot, rainy season and cool, dry seasons but has
considerably less annual rainfall. It is confined within the tropics and is best developed in Sudan, hence its
name the Sudan Climate. It is a transitional type of climate found between the equatorial rainforests and
hot deserts. Other features of this climate are:
1) Mean annual temperature is greater than 18° C.
2) Highest temperatures do not coincide with the period of the highest sun (e.g. June in the northern
hemisphere) but occur just before the onset of the rainy season, i.e. April in Northern Hemisphere and
October in Southern Hemisphere.
3) This extreme diurnal range of temperature is another characteristic feature of this climate with days
being hot and nights being cold.
4) The prevailing winds of the region are the Trade Winds, which bring rain to the coastal districts.
Source: 44.3D: Temperate Grasslands - Biology LibreTexts
Temperate Grasslands Biome: Location, Climate, Temperature, Plants and Animals | Conserve Energy
Future (conserve-energy-future.com)
What Are The Characteristics Of A Tropical Savanna Type Of Climate? - WorldAtlas

Q.39) Arrange the following surfaces in increasing order of their albedo:


1. Fresh snow
2. Sand
3. Grass
4. Thin Cloud
Select the correct answer using the code given below:
a) 3-4-2-1
b) 3-2-4-1
c) 2-3-4-1
d) 2-3-1-4

Ans) a
Exp) Option a is the correct answer.
Albedo is the portion of solar energy reflected from the surface of the Earth back into space.
It is a reflection coefficient and has a value of less than one.
Option a is correct.
When solar radiation passes through the atmosphere, a certain amount of it is scattered, reflected and
absorbed. The reflected sum of radiation is called the albedo of the earth.
Albedo is an important concept in climatology, astronomy, and environmental management.
It plays a major role in the energy balance of the earth’s surface, as it defines the rate of the absorbed
portion of the incident solar radiation
Different surfaces have different values.

Forum Learning Centre: Delhi - 2nd Floor, IAPL House, 19 Pusa Road, Karol Bagh, New Delhi - 110005 | Patna - 2nd floor, AG Palace, E Boring Canal Road,
Patna, Bihar 800001 | Hyderabad - 1st & 2nd Floor, SM Plaza, RTC X Rd, Indira Park Road, Jawahar Nagar, Hyderabad, Telangana 500020
9821711605 | https://academy.forumias.com | admissions@forumias.academy | helpdesk@forumias.academy
Page 31 of 39

SFG 2023 | LEVEL 1 | Test #27 – Solutions |


Albedo is higher in Snow or Ice.

Knowledge Base: NCERT: FUNDAMENTALS OF PHYSICAL GEOGRAPHY - chapter 9-SOLAR RADIATION,


HEAT BALANCE AND TEMPERATURE page 81

Q.40) With reference to the Pradhan Mantri Kisan Samridhi Kendra (PMKSK) Scheme, consider the
following statements:
1. It will help in providing agricultural inputs to the farmers.
2. It promotes competition among various fertiliser companies by ranking their brands according to its
quality.
3. It envisages the conversion of retail fertiliser shops into PMKSK.
Which of the statements given above is/are correct?
a) 1 and 3 only
b) 2 and 3 only
c) 1 only
d) 1, 2 and 3

Ans) a
Exp) Option a is the correct answer.
Recently, The Hon’ble Prime Minister Narendra Modi inaugurated 600 Pradhan Mantri Kisan Samruddhi
Kendra (PMKSK) under the Ministry of Chemicals & Fertilisers.
Statement 1 is correct: Under the scheme, the retail fertiliser shops in the country will be converted into
PMKSK in a phased manner. PMKSK will cater to a wide variety of needs of the farmers and provide Agri-
inputs (fertilisers, seeds, implements), testing facilities for soil, seeds, and fertilisers; generate
awareness among farmers.
Statement 2 is incorrect: Pradhan Mantri Bhartiya Jan Urvarak Pariyojana aims to make One Nation One
Fertilizer a reality. Under this scheme, the government of India launched Bharat Urea Bags and from now
on the urea sold in the country will be of the same name, same brand and same quality and it will be
available only under ‘Bharat’ brand name in the entire country (thus ranking brands based on its quality
is not correct)
Statement 3 is correct: The scheme ensures regular capacity building of retailers at block/district level
outlets. It also provides information regarding various government schemes. It envisages converting more
than 3.3 lakh retail fertiliser shops into PMKSK.
Source: https://pib.gov.in/PressReleasePage.aspx?PRID=1868496

Forum Learning Centre: Delhi - 2nd Floor, IAPL House, 19 Pusa Road, Karol Bagh, New Delhi - 110005 | Patna - 2nd floor, AG Palace, E Boring Canal Road,
Patna, Bihar 800001 | Hyderabad - 1st & 2nd Floor, SM Plaza, RTC X Rd, Indira Park Road, Jawahar Nagar, Hyderabad, Telangana 500020
9821711605 | https://academy.forumias.com | admissions@forumias.academy | helpdesk@forumias.academy
Page 32 of 39

SFG 2023 | LEVEL 1 | Test #27 – Solutions |


Q.41) The annual range of temperature in the interior of the continents is high as compared to coastal areas.
What is/are the reason/reasons?
1. Thermal difference between land and water
2. Variation in altitude between continents and oceans
3. Presence of strong winds in the interior
4. Heavy rains in the interior as compared to coasts
Select the correct answer using the codes given below.
a) 1 only
b) 1 and 2 only
c) 2 and 3 only
d) 1, 2, 3 and 4

Ans) a
Exp) Option a is the correct answer.
The annual range of temperature is defined as the difference between the hottest and coldest months at a
place, taking monthly mean temperatures in each case. It is given approximately by the difference between
the average of the January maximum and minimum temperatures, and the corresponding average for July.
Statement 1 is correct. The continents get heated faster and get cooled faster in comparison to the Oceans.
The annual range of temperature is high in the interior of the continent because places located in the
interior of the continent are far away from the moderating influence of the sea.
Statements 2 is incorrect. Altitude affects the daily range of temperature and annual mean temperature
but has negligible impact on annual range of temperature. Latitude affects the annual range of
temperature. The annual range of temperature increases with increasing latitude.
Statement 3 is incorrect. Wind is generally stronger near the coasts compared to interior areas of
continents.
Statement 4 is incorrect. Rainfall in the interiors of the Continents is generally low as compared to Coasts.
It is because the rain bearing winds from seas and oceans tend to loss most of the moisture till, they reach
the interiors.
The continents get heated faster and get cooled faster in comparison to the Oceans. The annual range of
temperature is high in the interior of the continent because places located in the interior of
the continent are far away from the moderating influence of the sea.
Source) UPSC 2013

Q.42) Which among the following are the likely reasons for Japan having a well-established fishing industry?
1. Meeting of warm and cold ocean currents near the coast of Japan
2. The presence of the indented coastline along the coasts of Japan.
3. Lack of plankton on continental shelves around the coasts of Japan.
Select the correct code from the options given below:
a) 1 and 3 only
b) 2 and 3 only
c) 1 and 2 only
d) 1, 2 and 3

Ans) c
Exp) Option c is the correct answer.
Commercial fishing is the activity of catching fish and other seafood for commercial profit, mostly from
wild fisheries. It provides a large quantity of food to many countries around the earth, but those who
practice it as an industry must often pursue fish far into the ocean under adverse conditions.

Forum Learning Centre: Delhi - 2nd Floor, IAPL House, 19 Pusa Road, Karol Bagh, New Delhi - 110005 | Patna - 2nd floor, AG Palace, E Boring Canal Road,
Patna, Bihar 800001 | Hyderabad - 1st & 2nd Floor, SM Plaza, RTC X Rd, Indira Park Road, Jawahar Nagar, Hyderabad, Telangana 500020
9821711605 | https://academy.forumias.com | admissions@forumias.academy | helpdesk@forumias.academy
Page 33 of 39

SFG 2023 | LEVEL 1 | Test #27 – Solutions |


Statement 1 is correct: The climate of Japan is modified by the meeting of warm and cold ocean currents.
It receives adequate rainfall from both the South-East Monsoon in summer and the North- West Monsoon
in winter (western coasts of Japan). The meeting zone between warm Kuroshio from the south and cold
Oyashio from the north produces fog and mist, making north Japan a ‘second Newfoundland’. Due to this
fishing becomes more favorable replacing agriculture as the main occupation in many of the indented
coastlands.
Statement 2 is correct: The indented coastline of Japan, provides sheltered fishing ports, calm waters, and
safe landing places, ideal for the fishing industry.
Statement 3 is incorrect: The continental shelves around the islands of Japan are rich in plankton, due to
the meeting of the warm Kuroshio and the cold Oyashio currents and provide excellent breeding grounds
for all kinds of fish.
Source: https://ncert.nic.in/ncerts/l/legy105.pdf
https://ncert.nic.in/ncerts/l/hess405.pdf

Q.43) With reference to the Tundra region, consider the following statements:
1. There is a general absence of tree cover in the tundra region.
2. Extraction and mining activities are totally prohibited in the Tundra region.
3. The Tundra plays a significant part in mitigating the negative impacts of carbon emission.
Which of the above statement is/are correct?
a) 1 and 2 only
b) 3 only
c) 1 and 3 only
d) 1, 2 and 3

Ans) c
Exp) Option c is the correct answer.
Tundra ecosystems are treeless regions found in the Arctic and on the tops of mountains, where the climate
is cold and windy, and rainfall is scant. Tundra lands are covered with snow for much of the year, but
summer brings bursts of wildflowers.
Statement 1 is correct: The tundra biome is a cold, treeless plain where harsh conditions make survival
difficult for both plants and animals. Tundra vegetation is composed of dwarf shrubs, sedges, grasses,
mosses, and lichens. Scattered trees grow in some tundra regions. The ecotone (or ecological boundary
region) between the tundra and the forest is known as the tree line or timberline. The tundra soil is rich in
nitrogen and phosphorus.
Statement 2 is incorrect: Human impact in the tundra biome is most obvious in the exploration and
development of mining, oil, gas, and other extractive industries.
There is no blanket ban on extraction and mining activities in the Tundra region around the World.
Statement 3 is correct: The Tundra plays a significant part in the planet's temperature management. It
works as a carbon sink, absorbing carbon dioxide from the atmosphere and assisting in the mitigation of
the negative impacts of carbon emissions.
Knowledge Base: Tundra climate is a Köppen climatic type with sub-freezing mean annual temperatures,
large annual temperature ranges (albeit not as large as the adjacent continental subarctic climate), and
moderately low precipitation. It is known for its frost-molded landscapes, extremely low temperatures,
little precipitation, depleted nutrients, and short growing seasons.
Source: https://sciencing.com/what-is-the-human-impact-on-the-tundra-13427779.html
https://education.nationalgeographic.org/resource/tundras-explained

Forum Learning Centre: Delhi - 2nd Floor, IAPL House, 19 Pusa Road, Karol Bagh, New Delhi - 110005 | Patna - 2nd floor, AG Palace, E Boring Canal Road,
Patna, Bihar 800001 | Hyderabad - 1st & 2nd Floor, SM Plaza, RTC X Rd, Indira Park Road, Jawahar Nagar, Hyderabad, Telangana 500020
9821711605 | https://academy.forumias.com | admissions@forumias.academy | helpdesk@forumias.academy
Page 34 of 39

SFG 2023 | LEVEL 1 | Test #27 – Solutions |


Q.44) Higher concentration of dust particles is found in subtropical and temperate regions as compared to
equatorial and polar regions. The most appropriate reason for this is:
a) Anti-cyclonic conditions in subtropical high-pressure belts
b) Presence of belt of strong upper-level wind circulation
c) Presence of drier winds in the region
d) Higher elevation of continental landmass in the region

Ans) c
Exp) Option c is the correct answer.
Dust is defined as sediment particles less than 100 µm in diameter. Dust can travel substantial distances
from continent to continent and across oceans and affects all of Earth's climatic zones from the tropics to
the poles. Atmosphere has a sufficient capacity to keep small solid particles, which may originate from
different sources and include sea salts, fine soil, smoke-soot, ash, pollen, dust and disintegrated particles
of meteors. Dust particles are generally concentrated in the lower layers of the atmosphere; yet,
convectional air currents may transport them to great heights.
The higher concentration of dust particles is found in subtropical and temperate regions due to dry
winds in comparison to equatorial and polar regions. Dust and salt particles act as hygroscopic nuclei
around which water vapour condenses to produce clouds. Polar regions are ice covered and so cannot be
dusty. Equatorial regions are either in oceans or there are dense equatorial rainforests in South America
and Africa. Hence there is very little dust there.
Option a is incorrect: Anticyclones are high-pressure systems and are more common in subtropical high-
pressure belts and polar high-pressure belts where the air is sinking from the upper troposphere to the
lower troposphere. The average velocity of an anticyclone is 30 to 50 km per hour. It is not the reason for
the higher concentration of dust particles found in subtropical and temperate regions.
Option b is incorrect: The subtropical high pressure is located at 30 degrees and 35 degrees both north
and south. The air in this region is comparatively dry and calm. It is not the reason for the higher
concentration of dust particles found in subtropical and temperate regions.
Option d is incorrect: The elevation of Continental landmass is not related to the higher concentration of
dust particles found in subtropical and temperate regions.
Source: NCERT 11th Geography Unit 4 Climate, Chapter 8 COMPOSITION AND STRUCTURE
OF ATMOSPHERE

Q.45) Which one of the following statements best reflects the definition of the term ‘Bhoota Kola’, that was
recently in news?
a) It is an ancient style of brass work prevalent in Western India.
b) It is a type of puppetry performed by certain tribal communities.
c) It is an ancient style of hand painting done on cotton clothes.
d) It is an annual folk ritual practised in some parts of Karnataka and Kerala.

Ans) d
Exp) Option d is the correct answer.
The traditional art Bhoota kola gained everyone's attention following the success of Kanada movie Kantara.
The film revolves around the tradition of a certain local community in Karnataka and portrays the art
Bhoota Kola.
Option d is correct: Bhoota Kola is an annual folk ritual of Tulu-speaking people in Dakshina Kannada,
Uttara Kannada and Udupi in Karnataka and some Kerala districts where local spirits or deities are
worshipped. It is performed by a trained person who is believed to have temporarily become a god himself.

Forum Learning Centre: Delhi - 2nd Floor, IAPL House, 19 Pusa Road, Karol Bagh, New Delhi - 110005 | Patna - 2nd floor, AG Palace, E Boring Canal Road,
Patna, Bihar 800001 | Hyderabad - 1st & 2nd Floor, SM Plaza, RTC X Rd, Indira Park Road, Jawahar Nagar, Hyderabad, Telangana 500020
9821711605 | https://academy.forumias.com | admissions@forumias.academy | helpdesk@forumias.academy
Page 35 of 39

SFG 2023 | LEVEL 1 | Test #27 – Solutions |


The face is painted, wrapped in a Siri made of coconut feathers, and danced invoking the deity. The Divine
Dancer gives justice to human beings and resolves disputes through the Word of God.
By praying together during Bhoota Kola, the community seeks God’s blessing, prosperity and riddance of
various problems the community is challenged with. It is said to have some influence from Yakshagana, a
more popular and widely performed folk dance in coastal Karnataka.
Source: https://karnatakatourism.org/destinations/bhootha-aradhane/
https://www.firstpost.com/explainers/explained-the-controversy-around-bhoota-kola-ritual-
depicted-in-kantara-11511171.html

Q.46) With reference to ‘Water Vapour in atmosphere’, consider the following statements:
1. It is responsible for about half of Earth’s greenhouse effect.
2. The amount of water vapour generally decreases from the equator towards the poles.
3. It redistributes heat energy on the Earth through latent heat energy exchange.
Which of the statements given above is/are correct?
a) 1 only
b) 1 and 2 only
c) 2 and 3 only
d) 1, 2 and 3

Ans) d
Exp) Option d is the correct answer.
Main gases of the atmosphere are nitrogen, oxygen, water vapour, carbon dioxide, methane, nitrous oxide,
and ozone. These gases are extremely important to the health of the Earth’s biosphere.
Statement 1 is correct: Water vapor is Earth’s most abundant greenhouse gas. It is responsible for about
half of Earth’s greenhouse effect — the process that occurs when gases in Earth’s atmosphere trap the
Sun’s heat. Greenhouse gases keep our planet liveable. It absorbs parts of the insolation from the sun and
preserves the earth’s radiated heat. It thus, acts like a blanket allowing the earth neither to become too
cold nor too hot.
Statement 2 is correct: Water vapour is a variable gas in the atmosphere, which decreases with altitude.
Water vapour decreases from the equator towards the poles. The highest concentrations of water vapour
are found near the equator over the oceans and tropical rain forests. Cold polar areas and subtropical
continental deserts are locations where the volume of water vapour can approach zero percent.
Statement 3 is correct: Water vapour has several very important functional roles on our planet. It
redistributes heat energy on the Earth through latent heat energy exchange. Heat radiated from Earth's
surface is absorbed by water vapor molecules in the lower atmosphere. The water vapor molecules, in turn,
radiate heat in all directions. Some of the heat returns to the Earth's surface. Thus, water vapor is a second
source of warmth (in addition to sunlight) at the Earth's surface.
Source: NCERT 11th Geography Unit 4 Climate, Chapter 8 COMPOSITION AND STRUCTURE
OF ATMOSPHERE
https://climate.nasa.gov/ask-nasa-climate/3143/steamy-relationships-how-atmospheric-water-vapor-
amplifies-earths-greenhouse-
effect/#:~:text=Water%20vapor%20is%20Earth's%20most,gases%20keep%20our%20planet%20livable.

Q.47) The thickness of the troposphere is greatest at the equator because of-
a) Convergence of northeast and the southeast trade winds near the equator.
b) Earth’s axis making an angle of 66½ with the plane of its orbit.
c) Heat being transported to great heights by strong convectional currents.
d) Bulging of the earth at equator as compare to poles

Forum Learning Centre: Delhi - 2nd Floor, IAPL House, 19 Pusa Road, Karol Bagh, New Delhi - 110005 | Patna - 2nd floor, AG Palace, E Boring Canal Road,
Patna, Bihar 800001 | Hyderabad - 1st & 2nd Floor, SM Plaza, RTC X Rd, Indira Park Road, Jawahar Nagar, Hyderabad, Telangana 500020
9821711605 | https://academy.forumias.com | admissions@forumias.academy | helpdesk@forumias.academy
Page 36 of 39

SFG 2023 | LEVEL 1 | Test #27 – Solutions |


Ans) c
Exp) Option c is the correct answer.
The troposphere is the lowermost layer of the atmosphere. Its average height is 13 km and extends roughly
to a height of 8 km near the poles and about 18 km at the equator. Thickness of the troposphere is greatest
at the equator because heat is transported to great heights by strong convectional currents. Heat
differential on the planet's surface causes convection currents to flow from the equator to the poles. This
implies that the warmer the weather, the thicker is the troposphere.

The troposphere is shallowest -- or narrowest -- at the poles, and deepest -- or thickest -- at the equator.
At the geographic North and South Poles, the troposphere reaches only 6 kilometers (4 miles) high, while
at the equator, it extends nearly 20 kilometers (12 miles) high.
Option a, b and d are incorrect: Convergence of northeast and south east trade winds, axial tilt of earth
and bulging of earth at equator are not directly responsible for the thickness of troposphere at equator.
Knowledge Base: NCERT 11th Geography Unit 4 Climate, Chapter 8 COMPOSITION AND STRUCTURE OF
ATMOSPHERE
https://education.seattlepi.com/over-part-earth-atmosphere-deepest-thickest-6169.html

Q.48) Consider the following pairs:


Atmospheric Features
Layers
1. Stratosphere Nearly free from
clouds and
weather
phenomena
2. Thermosphere Temperature
increases with
altitude
3. Mesosphere Aurora borealis is
observed here
How many pairs given above are correctly matched?
a) None of the pairs
b) Only one pair
c) Only two pairs
d) All three pairs

Ans) c
Exp) Option c is the correct answer.

Forum Learning Centre: Delhi - 2nd Floor, IAPL House, 19 Pusa Road, Karol Bagh, New Delhi - 110005 | Patna - 2nd floor, AG Palace, E Boring Canal Road,
Patna, Bihar 800001 | Hyderabad - 1st & 2nd Floor, SM Plaza, RTC X Rd, Indira Park Road, Jawahar Nagar, Hyderabad, Telangana 500020
9821711605 | https://academy.forumias.com | admissions@forumias.academy | helpdesk@forumias.academy
Page 37 of 39

SFG 2023 | LEVEL 1 | Test #27 – Solutions |


The atmosphere consists of different layers with varying density and temperature. Density is highest near
the surface of the earth and decreases with increasing altitude. The column of atmosphere is divided into
five different layers depending upon the temperature condition. They are: troposphere, stratosphere,
mesosphere, thermosphere and exosphere.
Pair 1 is correctly matched: Stratosphere is located between approximately 12 and 50 kilometres above
Earth’s surface. It is best known as home to Earth’s ozone layer, which protects us from the Sun’s harmful
ultraviolet radiation. Because of that UV radiation, the higher up you go into the stratosphere, the warmer
temperatures become. The stratosphere is nearly cloud- and weather-free, but polar stratospheric clouds
are sometimes present in its lowest, coldest altitudes. It’s also the highest part of the atmosphere that jet
planes can reach.
Pair 2 is correctly matched: Thermosphere is located between about 80 and 700 kilometres above Earth’s
surface, whose lowest part contains the ionosphere. In this layer, temperatures increase with altitude due
to the very low density of molecules found here. It is both cloud- and water vapor-free. The aurora borealis
and aurora australis are sometimes seen here. The International Space Station orbits in the thermosphere.
Pair 3 is incorrectly matched: Mesosphere is located between about 50 and 80 kilometre above Earth’s
surface, the mesosphere gets progressively colder with altitude. In fact, the top of this layer is the coldest
place found within the Earth system, with an average temperature of about minus 85 degrees Celsius (minus
120 degrees Fahrenheit). The very scarce water vapor present at the top of the mesosphere forms
noctilucent clouds, the highest clouds in Earth’s atmosphere, which can be seen by the naked eye under
certain conditions and at certain times of day. Most meteors burn up in this atmospheric layer. Sounding
rockets and rocket-powered aircraft can reach the mesosphere.
Most Earth satellites orbit in the exosphere and aurora borealis and aurora australis are seen in
thermosphere.

Source: https://climate.nasa.gov/news/2919/earths-atmosphere-a-multi-layered-cake/
NCERT 11th Geography Unit 4 Climate, Chapter 8 COMPOSITION AND STRUCTURE OF ATMOSPHERE

Q.49) With reference to ‘Solar Radiation on Earth’, consider the following statements:
1. The surface of Earth receives most of its energy in long ultraviolet wavelength.
2. The solar output received at the top of the atmosphere is constant throughout the year.
3. The angle of inclination of the rays of sun influences the amount of insolation received at different
latitudes.

Forum Learning Centre: Delhi - 2nd Floor, IAPL House, 19 Pusa Road, Karol Bagh, New Delhi - 110005 | Patna - 2nd floor, AG Palace, E Boring Canal Road,
Patna, Bihar 800001 | Hyderabad - 1st & 2nd Floor, SM Plaza, RTC X Rd, Indira Park Road, Jawahar Nagar, Hyderabad, Telangana 500020
9821711605 | https://academy.forumias.com | admissions@forumias.academy | helpdesk@forumias.academy
Page 38 of 39

SFG 2023 | LEVEL 1 | Test #27 – Solutions |


Which of the statements given above is/are correct?
a) 1 only
b) 2 only
c) 2 and 3 only
d) 3 only

Ans) d
Exp) Option d is the correct answer.
Statement 1 is incorrect: The earth-atmosphere energy balance is the balance between incoming energy
from the Sun and outgoing energy from the Earth. Energy released from the Sun is emitted as shortwave
light and ultraviolet energy. The earth’s surface receives most of its energy in short wavelengths.
Statement 2 is incorrect: The solar output received at the top of the atmosphere varies slightly in a year
due to the variations in the distance between the earth and the sun. During its revolution around the sun,
the earth is farthest from the sun (152 million km) on 4th July. This position of the earth is called aphelion.
On 3rd January, the earth is the nearest to the sun (147 million km). This position is called perihelion.
Therefore, the annual insolation received by the earth on 3rd January is slightly more than the amount
received on 4th July.
Statement 3 is correct: The amount and the intensity of insolation vary during a day, in a season and in a
year. The factors that cause these variations in insolation includes the angle of inclination of the sun’s
rays. The fact that the earth’s axis makes an angle of 66½ with the plane of its orbit round the sun has a
greater influence on the amount of insolation received at different latitudes. The higher the latitude the
less is the angle they make with the surface of the earth resulting in slant sun rays. The area covered by
vertical rays is always less than the slant rays. If more area is covered, the energy gets distributed and the
net energy received per unit area decreases. Moreover, the slant rays are required to pass through greater
depth of the atmosphere resulting in more absorption, scattering and diffusion.

Source: NCERT 11th Geography Unit 4 Climate, Chapter 9 SOLAR RADIATION, HEAT BALANCE
AND TEMPERATURE

Q.50) With reference to the Law Commission of India, consider the following statements:
1. It is a non-constitutional and non-statutory body.
2. Its recommendations are binding on the Government.
3. It can be chaired only by a retired judge of the Supreme court.
4. Its origin can be traced back to the pre-independent India.
Which of the statements given above is/are correct?
a) 1 only
b) 1, 3 and 4 only

Forum Learning Centre: Delhi - 2nd Floor, IAPL House, 19 Pusa Road, Karol Bagh, New Delhi - 110005 | Patna - 2nd floor, AG Palace, E Boring Canal Road,
Patna, Bihar 800001 | Hyderabad - 1st & 2nd Floor, SM Plaza, RTC X Rd, Indira Park Road, Jawahar Nagar, Hyderabad, Telangana 500020
9821711605 | https://academy.forumias.com | admissions@forumias.academy | helpdesk@forumias.academy
Page 39 of 39

SFG 2023 | LEVEL 1 | Test #27 – Solutions |


c) 2 and 4 only
d) 1 and 4 only

Ans) d
Exp) Option d is the correct answer.
Recently, the Supreme Court issued notice to the Centre on PIL seeking the Law Commission to be
declared a ‘statutory body'. The PIL contended that the non-working of the Law Commission is depriving
the Centre of the benefit of the law commission’s insights on the various aspects of the law.
Statement 1 is correct: Law Commission of India is a non-constitutional and non-statutory body and is
constituted by a notification of the Government of India to carry out research in the field of law and make
recommendations to the Government.
Statement 2 is incorrect: The recommendations of the law commission are not binding on the
government even the recommendations were made on the matter referred to it by the Supreme court. The
law commission has taken up various subjects on references made by the Department of Legal Affairs,
Supreme Court and High Courts and submitted 277 reports.
Statement 3 is incorrect: The retired chief justice of the High court can also become a chairperson of
the Law commission. Retired High Court Chief Justice Rituraj Awasthi has been appointed as the
chairperson of the 22nd law commission of India which was constituted in 2020. Also Mr. M. C. Setalvad
who is the first chairperson of the law commission was an Attorney General of India.
Statement 4 is correct: The first Law Commission was established during the British Raj era in 1834. It
was established by the Charter Act of 1833 and was chaired by Lord Macaulay. In 1955, the first Law
Commission of independent India was established for a three-year term. Since then, twenty-one more
Commissions have been established
Source:
https://lawcommissionofindia.nic.in/#:~:text=Law%20Commission%20of%20India%20is,Government%
20(in%20the%20form%20of
https://indianexpress.com/article/india/sc-notice-to-centre-on-pil-seeking-law-commission-to-be-
declared-statutory-body-7161881/

Forum Learning Centre: Delhi - 2nd Floor, IAPL House, 19 Pusa Road, Karol Bagh, New Delhi - 110005 | Patna - 2nd floor, AG Palace, E Boring Canal Road,
Patna, Bihar 800001 | Hyderabad - 1st & 2nd Floor, SM Plaza, RTC X Rd, Indira Park Road, Jawahar Nagar, Hyderabad, Telangana 500020
9821711605 | https://academy.forumias.com | admissions@forumias.academy | helpdesk@forumias.academy

You might also like